Anda di halaman 1dari 70

1

CNS
Define stroke. Discuss pathophysiology, investigation and
management of cerebral infarction.Define stroke. Discuss the
etiopathogenesis, clinical features of stroke. Outline the
management of hemorrhagic stroke. Paper II aug 2011
Definition: WHO
Stroke:
A focal or global neurological impairment of sudden onset, lasting more
than 24 hours (or leading to death), of vascular origin.
Progressing stroke (or stroke in evolution):
This describes a stroke in which the focal neurologicaldeficit worsens after
the patient first presents due to increasing infarction, haemorrhagic
transformation or oedema.
Completed stroke:
This describes a stroke in which the focal deficit persists and is not
progressing.
TIA:
Transient ischemic attack is the acute onset of a focal neurologic deficit in
a specific vascular distribution with full recovery within 24 hours of onset .
Reversible Ischemic Neurological Deficit (RIND)
A focal brain ischemia in which the deficit improves over a maximum of 72
hours
Etiology of stroke:
1. Ischemic stroke:
1. Atherosclerosis of carotid artery and aortic arch.
2. Embolism from the heart (eg. Atrial fibrillation)
3. Intrinsic disease of lenticulostriate arteries producing lacunar
infarctions.
4. Vasculitis,
5. Endocarditis
6. Cerebral venous thromosis (eg. dehydration)
2. Haemorrhagic stroke:
1. Intracerebral
2. Suarachnoid
ISCHEMIC STROKE
1. The risk factors for ischaemic stroke:
1. Fixed
1. More common in old age
2. Gender (male > female)
3. Race (Afro-Caribbean > Asian > European)
4. Heredity
5. Previous vascular event, e.g. myocardial infarction, stroke or
peripheral embolism
6. High fibrinogen
2. Modifiable
1. High blood pressure
2. Heart disease (atrial fibrillation, heart failure, endocarditis)
3. Diabetes mellitus

2
4. Hyperlipidaemia
5. Smoking
6. Excess alcohol consumption
7. Polycythaemia
8. Oral contraceptives
9. Social deprivation
3. About 5% are due to rare causes, including
1. Vasculitis
2. Endocarditis
3. Cerebral venous disease
2. Pathogenesis:
1. After the occlusion of a cerebral artery, compensatory mechanisms
come into play:
1. Opening of anastomotic channels may try to restore perfusion
2. Vasodilatation
3. Tissue oxygen extraction
2. If the cerebral blood flow is not compensated:
1. Reduction of blood flow reduces supply of oxygen and hence ATP.
2. H+ is produced by anaerobic metabolism and consequent fall in
tissue pH
3. cytotoxic oedema and membrane depolarisation, allowing
calcium entry and releasing glutamate.
4. Calcium enters cells activates destructive intracellular enzymes,
destroying intracellular organelles and cell membrane, with
release of free radicals.
5. Free fatty acid release activates pro-coagulant pathways which
exacerbate local ischaemia.
6. The release of inflammatory mediators by microglia and
astrocytes produces death of all cell types in the area of
maximum ischaemia.
3. Subsequent restoration of blood flow may cause haemorrhage into the
infracted area (haemorrhagic transformation).
4. Radiologically, a cerebral infarct can be seen as a mixture of dead brain
tissue that is already undergoing autolysis and tissue that is ischaemic
and swollen but recoverable (the ischaemic penumbra).
5. As the weeks go by, the oedema subsides and the infarcted area is
replaced by fluid-filled cavity.
3. Clinical features
1. The clinical presentation of stroke depends upon arterial territory
2. Cerebral hemisphere: unilateral motor deficit, loss of higher cerebral
function such as aphasia, visual field defect
3. Brain stem or Cerebellum: Ataxia, diplopia, vertigo and/or bilateral
weakness
4. Middle cerebral artery: arm more severely affected than the leg;
dominant hemispherical involvement produces motor aphasia.
5. Anterior cerebral artery: leg is more affected than the arm.
6. Posterior cerebral: homomynous hemianopsia.
7. Basilar artery: vertigo, diplopia, dysarthria or Horner syndrome;
hemiparesis is not a feature.
4. General examination: for comorbidities
Skin
1. Xanthelasma

3
2. Rash (arteritis, splinter haemorrhages, livedo reticularis)
3. Colour and temperature change (limb ischaemia/deep
venous thrombosis)
Eyes
Arcus senilis
Diabetic changes
Hypertensive changes
Retinal emboli
Cardiovascular system
1. Heart rhythm (atrial fibrillation)
2. Blood pressure (hypertension, hypotension)
3. Jugular venous pressure (heart failure, hypovolaemia)
4. Murmurs (sources of embolism)
5. Peripheral pulses and bruits (generalised arteriopathy)
Respiratory system
1. Signs of pulmonary oedema
2. Signs of respiratory infection
Abdomen
1. Palpable bladder (urinary retention)
Locomotor
1. Injuries sustained during collapse with stroke
Investigations
Cause
Investigation
Cardiac embolism
Echocardiography (including
transoesophageal)
Premature atherosclerosis
Serum lipids
Arterial dissection
MRI Angiography
Thrombophilia
Protein C, protein S Antithrombin III Factor
V Leiden, prothrombin
Homocystinuria
Urinary amino acids Methionine loading
test
Antiphospholipid antibody
Anticardiolipin antibodies/lupus
anticoagulant
syndrome
SLE
Antinuclear antibodies
Vasculitis
ESR CRP Antineutrophil cytoplasmic
antibody
1.
2.
3.
4.

4.

CADASIL (cerebral autosomal


biopsy
dominant arteriopathy with
subcortical infarcts and
leucoencephalopathy)
Mitochondrial cytopathy

Fabrys disease
Neurovascular syphilis
5. Investigation of risk factors

MRI brain Genetic analysis Skin

Serum lactate White cell mitochondrial


DNA Muscle biopsy; Mitochondrial
molecular genetics
Alpha-galactosidase levels in blood
Syphilis serology

4
Vascular lesion:

CT/MRI Duplex ultrasound of carotids


Magnetic resonance angiography (MRA)
CT angiography (CTA) Contrast
angiography
Subarachnoid haemorrhage:
CT/lumbar puncture
Cardiac source of embolism:
Electrocardiogram; Echocardiogram

6.

7.

8.

9.

Routine evaluation:
Full blood count
Cholesterol
Blood glucose
ESR
Serum protein electrophoresis
Clotting/thrombophilia screen
Neuroimaging
1. CT Brain:
a. Demonstrates subdural haematomas and brain tumours,
intracerebral haemorrhage
b. Abnormal perfusion of brain tissue can be imaged with CT after
injection of contrast media (i.e. perfusion scanning).
2. MRI:
a. MRI diffusion weighted imaging (DWI) can detect ischaemia
earlier than CT, and other
b. MRI sequences can also be used to demonstrate abnormal
perfusion.
c. It can reliably distinguish haemorrhagic from ischaemic stroke
even several weeks after the onset.
3. CT and MRI may also reveal lacunar infarct indicating small-vessel
disease; It might indicate the presence of an underlying vascular
malformation, saccular aneurysm or amyloid angiopathy.
Vascular imaging
1. Carotid arterial disease can be non-invasively identified with duplex
ultrasound, MR angiography (MRA) or CT angiography.
2. intra-arterial contrast angiography is reserved for patients to delineate
a saccular aneurysm, an arteriovenous malformation or vasculitis.
Cardiac investigations
1. ECG: To idendify atrial fibrillation, prosthetic heart valves, other valvular
abnormalities and recent myocardial infarction.
2. A transthoracic or transoesophageal echocardiogram can confirm the
presence of endocarditis, atrial myxoma, intracardiac thrombus or
patent foramen ovale.
Management
1. Management is aimed at minimising the brain damage, preventing
complications, reducing the patients disability and handicap through
rehabilitation, and reducing the risk of recurrent episodes.
2. Dysphagia: nasogastric tube or intravenous feeding.
3. Deteriorating stroke as a result of complications such as hypoxia,
sepsis, epileptic seizures or metabolic abnormalities are managed
oxygen, antibiotics and antiepileptics and IV fluid corrections

5
4. Anti-oedema agents, such as mannitol or artificial ventilation, and
surgical decompression to reduce intracranial pressure should be
considered in appropriate patients.
5. Obstructive hydrocephalus and will benefit from insertion of a
ventricular drain and/or decompressive surgery.
6. Thrombolysis: Intravenous thrombolysis with recombinant tissue
plasminogen activator (rt-PA) is given within 3 hours of symptom onset
that minimises the haemorrhagic risk
7. Aspirin: aspirin (300 mg daily) should be started immediately after an
ischaemic stroke unless rt-PA has been given, in which case it should be
withheld for at least 24 hours.
8. Heparin: Anticoagulation with heparin has been widely used to treat
acute ischaemic stroke but these benefits are offset by a definite
increase in the risk of both intracranial and extracranial haemorrhage.
10. Management of risk factors
1. long-term antiplatelet drugs and statins to lower cholesterol.
2. For atrial fibrillation the risk can be reduced by oral anticoagulation to
achieve an INR of 23.
3. Control of blood pressure
11.
Carotid endarterectomy and angioplasty:
a. Endarterectomy of carotid stenosis has been shown to reduce the
subsequent risk of stroke, but the small absolute benefit does not justify
its routine use.
b. Carotid angioplasty and stenting are technically feasible but have not
been shown to be superior to endarterectomy.
HAEMORRHAGIC STROKE
1. Intracerebral haemorrhage
1. Rupture of a blood vessel within the brain parenchyma
2. Haemorrhage can occur into an area of brain infarction.
3. If big enough, this can cause shift of the intracranial contents,
producing transtentorial coning and sometimes rapid death.
4. If the patient survives, the haematoma is gradually absorbed,
leaving a haemosiderin-lined slit in the brain parenchyma.
2. The risk factors for haemorrhagic stroke:
1. Complex small vessel disease with disruption of vessel
wall
1. Old age
2. Hypertension
3. Low cholesterol
2. Amyloid angiopathy
3. Impaired blood clotting
1. Anticoagulant therapy
2. Blood dyscrasia
3. Thrombolytic therapy
4. Vascular anomaly
1. Arteriovenous malformation
2. Cavernous haemangioma
5. Substance misuse
1. Alcohol
2. Amphetamines
3. Cocaine

6
3. Types of haemorrhgic stroke:
1. Hemorrhagic transformation of ischemic stroke
2. Subarachnoid hemorrhage: Rupture of an intracranial
(1) Berry aneurysm (EhlersDanlos syndrome.)

(2)

AV Malformation
3. Spontaneous Intracerebral hemorrhage :
(1)Capillary or venous rupture (hypertension)
(2) Hematologic and bleeding disorders (eg, leukemia,
thrombocytopenia, hemophilia, or disseminated
intravascular coagulation),
(3) anticoagulant therapy,
(4) liver disease,
(5) high alcohol intake, and
(6) primary or secondary brain tumors.
4. Clinical features
1. Physical exertion, straining and sexual excitement are common
antecedents.
2. Subarachnoid haemorrhage typically presents with a sudden,
severe thunderclap headache (often occipital) often
accompanied by vomiting.
3. There may be loss of consciousness at the onset, so
subarachnoid haemorrhage should be considered if a patient is
found comatose.
4. On examination the patient is usually distressed and irritable,
with photophobia.
5. There may be neck stiffness due to subarachnoid blood
6. Focal hemisphere signs such as hemiparesis or aphasia may be
present at onset
7. A third nerve palsy may be present due to local pressure from an
aneurysm of the posterior communicating artery, though this is
rare.
8. Fundoscopy may reveal a subhyaloid haemorrhage, which
represents blood tracking along the subarachnoid space around
the optic nerve.
5. Investigations:
1. Spontaneous intracerebral hemorrhage:
(1) Non contrast CT is superior to MRI for detecting bleeds of
< 48 hours duration;
(2) Laboratory tests to identify bleeding disorder such as
bleeding and clotting and prothrombin time
(3) angiography may be indicated to exclude aneurysm or
AVM.
(4) Do not perform lumbar puncture.
2. Subarachnoid hemorrhage:
(1) CT to confirm diagnosis, but may be normal in rare
instances;
(2) lumbar puncture to look for red blood cells or
xanthochromia;
(3) Angiography to determine source of bleed in candidates
for treatment.
3. Intracranial aneurysm:

7
(1)CT indicates subarachnoid hemorrhage, and angiography
then demonstrates aneurysms;
(2)Angiography may not reveal aneurysm if vasospasm
present.
6. Management
1. Stroke is managed as in ischemic stroke
2. Aneurysm: Insertion of platinum coils into an aneurysm (via an
endovascular procedure) or surgical clipping of the aneurysm
neck reduces the risk of both early and late recurrence.
3. Arteriovenous malformations can be managed in several
different ways including surgical removal, ligation of the blood
vessels that feed or drain the lesion, or injection of material to
occlude the fistula or draining veins.
4. Nimodipine (3060 mg i.v. for 514 days, followed by 360 mg
orally for a further 7 days) is usually given to prevent vasospasm
in the acute phase.
5. Treatment may also be required for complications of SAH, which
include:
(1)obstructive hydrocephalus (possibly requiring drainage via
a shunt),
(2)delayed cerebral ischaemia due to vasospasm
(vasodilators),
(3)hyponatraemia (water restriction) and
(4)systemic complications associated with immobility, such
as chest infection and venous thrombosis.

EPILEPSY AND STATUS EPILEPTICUS. AUG 2012 PAPER II FEB 2011


Classification of epilepsy
By Seizure type
1. Partial (simple or complex)
2. Partial with secondary generalisation
3. Absence
4. Tonic clonic
5. Tonic
6. Atonic
7. Myoclonic
By Physiology (EEG)
1. Focal - spikes/sharp waves
2. Generalised - spike and wave
By Anatomical site
1. Cortex
a. Temporal
b. Frontal
c. Parietal
d. Occipital
2. Generalised (diencephalon)
3. Multifocal
By Pathological causes
1. Genetic
2. Developmental

8
3. Tumours
4. Trauma
5. Vascular
6. Infections
7. Inflammation
8. Metabolic
9. Drugs, alcohol and toxins
10. Degenerative

Etiology:
Causes of secondary generalised epilepsy
1. Genetic
a. Inborn errors of metabolism
b. Storage diseases
c. Phakomatoses (e.g. tuberous sclerosis)
2. Cerebral birth injury
3. Hydrocephalus
4. Cerebral anoxia
5. Drugs
a. Antibiotics: penicillin, isoniazid, metronidazole
b. Antimalarials: chloroquine, mefloquine
c. Cyclosporin
d. Cardiac anti-arrhythmics: lidocaine, disopyramide
e. Psychotropic agents: phenothiazines, tricyclic antidepressants,
lithium
f. Amphetamines (withdrawal)
6. Alcohol (especially withdrawal)
7. Toxins
a. Heavy metals (lead, tin)
b. Organophosphates (sarin)
8. Metabolic disease
a. Hypocalcaemia
b. Hypoglycaemia
c. Hyponatraemia
d. Renal failure
e. Hypomagnesaemia
f. Liver failure
9. Infective
a. Meningitis
b. Post-infectious encephalopathy
10. Inflammatory
a. Multiple sclerosis (uncommon)
b. SLE
11. Diffuse degenerative diseases
a. Alzheimers disease (uncommonly)
b. CreutzfeldtJakob disease (rarely)
4. Pathophysiology
1. Seizures can be viewed as resulting from an imbalance between
excitatory and inhibitory processes in the brain
2. Decreased inhibitory neurotransmission which is primarily by gammaamino butyric acid (GABA), or enhanced excitatory neurotransmission
which is primarily mediated by glutamate.
3. Small group of abnormal neurons undergo prolonged depolarizations
associated with the rapid firing of repeated action potentials.

9
4. Epileptic neurons recruit adjacent neurons creating a storm of electrical
activity in the brain leading to clinical seizure
5. Partial seizure originates from a paroxysmal discharge in a focal area
and subsequently spread to the rest of the brain (secondary
generalisation)
Common Epilepsy Triggers:
1. Flickering or Flashing Light
2. Hyperventilation
3. Lack of Sleep
4. During ovulation, menstruation, pregnancy or menopause. This is
known as catamenial epilepsy.
5. Especially foods that are rich in glutamate and aspartame
6. High fever
7. Hot water- bath epilepsy
8. TV epilepsy

Clinical features:
Typical progression of tonic clonic seizure:
1.
The prodrome: change in mood or behaviour or
insomnia or feelings about the impending seizure
1. The aura: feeling a sense of unexplained fear, a strange smell or a
feeling of numbness; dj vu phenomenon
3. Epileptic cry: Some people may cry out at the beginning of a seizure as
the muscles around the vocal cords contract
4. Loss of consciousness
5. Tonic phase:
6. Clonic phase:
7. Loss of bowel and bladder control.
8. Postictal confusion .
9. Postictal automatism
10.Post ictal sleep
11.Somnambulism
12.Severe headache.
13.Todds palsy
Seizure types:
Complex partial seizures: eg psychomotor epilepsy
1. It involves episodes of altered consciousness without the patient
collapsing to the ground
2. Patients stop what they are doing and stare blankly, often blinking
repetitively, making smacking movements of their lips, or displaying
other automatisms, such as picking at their clothes.
3. After a few minutes, consciousness returns but the patient may be
muddled and feel drowsy for a period of up to an hour.
4. Immediately before such an attack the patient may report alteration of
mood, memory or perception (such as undue familiaritydj vuor
unrealityjamais vu), or complex hallucinations involving sound, smell,
taste, vision, emotion (fear, sexual arousal) or visceral sensations
(nausea, epigastric discomfort).
Absence seizures (petit mal):

10
1. It always starts in childhood. The attacks are of shorter in duration;
they occur much more frequently (2030 times a day) and are not
associated with post-ictal confusion.
2. Absence attacks are caused by a generalised discharge that does not
spread out of the hemispheres and so does not cause loss of posture.
Atonic seizures
1. These are seizures involving brief loss of muscle tone, usually resulting
in heavy falls with or without loss of consciousness.
2. They only occur in the context of epilepsy syndromes which involve
other forms of seizure.
Partial motor seizures
1. Epileptic activity arising in the pre-central gyrus causes partial motor
seizures affecting the contralateral face, arm, trunk or leg. They may
remain localised to one part, or may spread to involve the whole side.
Some attacks begin in one part of the body (e.g. mouth, thumb, great
toe) and spread (march) gradually to other parts of the body; this is
known as a Jacksonian seizure.
2. Attacks vary in duration from a few seconds to several hours (epilepsia
partialis continua).
3. More prolonged episodes may be followed by paresis of the involved
limb lasting for several hours after the seizure ceases (Todds palsy).
Partial sensory seizures
1. Seizures arising in the sensory cortex cause unpleasant tingling or
electric sensations in the contralateral face or limbs.
2. A spreading pattern similar to a Jacksonian seizure may occur, the
abnormal sensation spreading over the body much faster (in seconds)
than the march of a migrainous focal sensory attack, which spreads
over 2030 minutes.
Versive seizures
1. A frontal epileptic focus may involve the frontal eye field, causing
forced deviation of the eyes and sometimes turning of the head to the
opposite side.
2. This type of attack often generalises to become a tonic clonic seizure.
Partial visual seizures
1. Occipital epileptic foci cause simple visual hallucinations such as balls
of light or patterns of colour.
2. Formed visual hallucinations of faces or scenes arise more anteriorly in
the temporal lobes.

STATUS EPILEPTICUS
1. Status epilepticus is defined as a seizure or a series of seizures lasting 30
minutes without the patient regaining awareness between attacks.
2. It may be precipitated by abrupt withdrawal of anticonvulsant drugs, the
presence of a major structural lesion or acute metabolic disturbance.
3. It tends to be more common with frontal epileptic foci.
4. Management
a. Initial
i. Ensure airway is patent, give oxygen to prevent cerebral
hypoxia, and secure intravenous access
ii. Draw blood for glucose, urea and electrolytes (including Ca
and Mg), and liver function, and store a sample for future

11
analysis (e.g. drug misuse)
iii. Give diazepam 10 mg i.v. (or rectally) or lorazepam 4 mgi.v.
repeat once only after 15 mins
iv. Transfer to intensive care area, monitoring neurological
condition, blood pressure, respiration and blood gases,
intubating and ventilating patient if appropriate
b. Ongoing
i. If seizures continue after 30 mins
1. I.v. infusion (with cardiac monitoring) with one of:
a. Phenytoin: 15 mg/kg at 50 mg/min
b. Fosphenytoin: 15 mg/kg at 100 mg/min
c. Phenobarbital: 10 mg/kg at 100 mg/min
ii. If seizures still continue after 3060 mins
1. Start treatment for refractory status with intubation,
ventilation, and general anaesthesia using propofol or
thiopental
iii. Once status controlled
1. Commence longer-term anticonvulsant medication with
one of:
a. Sodium valproate 10 mg/kg i.v. over 35 mins,
then8002000 mg/day
b. Phenytoin: give loading dose (if not already
usedas above) of 15 mg/kg, infuse at < 50
mg/min, then 300 mg/day Carbamazepine 400
mg by nasogastric tube, then 4001200 mg/day
iv. Investigate cause

PARKINSONISM FEB 2013; .PII AUG 2009; : FEB 2009; AUG 2008
PII
1. Parkinsonism is a general term that is used to define a symptom complex
manifest by bradykinesia with rigidity and/or tremor. It reflects damage to
different components of the basal ganglia.
2. The basal ganglia comprise a group of subcortical nuclei that include the
striatum (putamen and caudate nucleus), subthalamic nucleus (STN),
globus pallidus pars externa (GPe), globus pallidus pars interna (GPi), and
the SNc (substantia nigra pars compacta)
Etiology:
1. Parkinson's Disease:
1. Genetic
2. Sporadic
3. Dementia with Lewy bodies
2. Secondary Parkinsonism:
1. Drug-induced
2. Tumor
3. Infection
4. Vascular
5. Normal-pressure hydrocephalus
6. Trauma
7. Liver failure
8. Toxins (e.g., carbon monoxide, manganese, MPTP, cyanide, hexane, methanol,
carbon disulfide)
3. Neurodegenerative Disorders:
1. Wilson's disease

12
2. Huntington's disease
3. Alzheimer's disease with parkinsonism

Pathophysiology
1. A small number of cases are familial in nature and mutations in several
genes; in the majority no strong genetic factors have been identified.
2. Methyl-phenyl-tetrahydropyridine (MPTP) caused severe parkinsonism in
young drug users suggesting an environmental toxin;
3. Pathology:
a. Depletion of the pigmented dopaminergic neurons in the substantia
nigra, hyaline inclusions in nigral cells (Lewy bodies), atrophic
changes in the substantia nigra and depletion of neurons in the
locus coeruleus.
b. Reduced dopaminergic output from the substantia nigra to the
globus pallidus. This in turn results in bradykinesia.
Clinical features
1. General
a. Expressionless face
b. Greasy skin
c. Soft, rapid, indistinct speech
d. Flexed posture
e. Impaired postural reflexes
2. Gait
a. Slow to start walking
b. Shortened stride
c. Rapid, small stride length, tendency to shorten (festination)
d. Reduced arm swing
e. Impaired balance on turning
3. Tremor
a. Resting (46 Hz)
i. Coarse, complex movements, usually first in fingers/thumb
ii. Flexion/extension of fingers
iii. Abduction/adduction of thumb
iv. Supination/pronation of forearm
v. May affect arms, legs, feet, jaw, tongue
vi. Intermittent, present at rest and when distracted
vii. Diminished on action
b. Postural (810 Hz)
i. Less obvious, faster, finer amplitude
ii. Present on action or posture, persists with movement
4. Rigidity
a. Cogwheel type, mostly upper limbs
b. Plastic (lead pipe) type, mostly legs
5. Bradykinesia
a. Slowness in initiating or repeating movements
b. Impaired fine movements, especially of fingers
6. Although the features are initially unilateral, gradual bilateral involvement
is the rule. Muscle strength and reflexes remain normal, and plantar
responses are flexor.
7. There is a paucity of facial expression (hypomimia) and the blink reflex
may be exaggerated and fail to habituate (glabellar tap sign).
8. Eye movements are normal to standard clinical testing, provided allowance
is made for the normal limitation of upward gaze with age.
9. Sensation is normal and intellectual faculties are not affected initially.

13
10.As the disease progresses, about one-third of patients develop cognitive
impairment.
Investigations
1. The diagnosis is made clinically, as there is no diagnostic test for Parkinsons
disease.
2. Sometimes it is necessary to investigate patients to exclude other causes of
parkinsonism if there are any unusual features.
3. Patients\presenting before the age of 50 are usually tested for Wilsons
disease,

Medical management
1. Levodopa.
a. The initial dose is 50 mg 8- or 12-hourly, increased if necessary. The

total levodopa dose may be increased to over 1000 mg/day if


necessary.
b. Side-effects:
1. postural hypotension, nausea and vomiting
2. depression, hallucinations and delusions.
3. change in personality with increased gambling, hypersexuality
4. End-of-dose deterioration due to progression of the disease and
loss of capacity to store dopamine.
5. Alternate dyskinesia and agitation (the on-off phenomenon).
Subcutaneous injections of apomorphine (a dopamine agonist)
are helpful to rescue the patient rapidly from an off period.
2. Carbidopa and benserazide:
a. are decarboxylase inhibitors, and available as combination preparations
with levodopa ( Sinemet and Madopar)
b. They inhibit peripheral decarboxylation of dopamine and make it
available to brain
3. Anticholinergic agents.
a. These have a useful effect on tremor and rigidity, but do not help
bradykinesia.
b. Trihexyphenidyl (benzhexol; 14 mg 8-hourly) and orphenadrine (50
100 mg 8-hourly).
4. Dopamine receptor agonists.
a. Apomorphine given alone causes marked vomiting and has to be
administered parenterally. The vomiting can be overcome by the
concomitant use of domperidone,
b. Bromocriptine is 1 mg initially, increased to 2.5 mg 8-hourly, and
thereafter up to 30 mg/day.
5. Dopamine reuptake inhibitor
a. Amantadine. useful in controlling the dyskinesias produced by
dopaminergic treatment later in the disease. The dose is 100 mg 8- or
12-hourly. Sideeffects include livedo reticularis, peripheral oedema,
confusion and seizures.
6. MAO-B inhibitor
a. Selegiline. It slows the progression of the disease; it might be

associated with an increased risk of sudden death. The usual dose is 5


10 mg in the morning.
7. COMT (catechol-O-methyl-transferase) inhibitors

14
a. Entacapone (200 mg with each dose of levodopa) prolongs the effects

of each dose and reduces motor fluctuations when used with levodopa.
This allows the levodopa dose to be reduced and given less frequently.
Surgery
1. Stereotactic thalamotomy can be used to treat tremor,
2. Implantation of stimulating electrodes into the globus pallidus to help in

the management of drug-induced dyskinesia.


3. The implantation of fetal mid-brain cells into the basal ganglia to

enhance dopaminergic activity remains experimental.


Physiotherapy and speech therapy
1. Patients at all stages of Parkinsons disease benefit from physiotherapy,
which helps reduce rigidity and corrects abnormal posture.
2. Speech therapy may help in patients where dysarthria and dysphonia
interfere with communication.

GUILLAIN BARRE SYNDROME. PAPER II FEB 2010; AUG 2009 PII


Etiology:
1. This syndrome of acute paralysis develops, in 70% of patients, 14 weeks
after infection:
a. GIT: camphylobacter jejuni, Helicobacter,
b. R.S: Mycoplasma pneumonia
c. Vaccines: Rabies, influenza, oral polio and meningococcal and H1N1
vaccines.
Pathogenesis:
1. Resemblance of the pathogens to antigens on peripheral nerves
(molecular mimicry) leads to an overzealous autoimmune response
mounted by T-lymphocytes and macrophages
2. Autoimmune process leads to:
i. Demyelination
ii. Axonal degenerataion
Clinical features
1. Distal paraesthesia and limb pains (often severe) precede a rapidly
ascending muscle weakness, from lower to upper limbs, more marked
proximally than distally.
2. Facial and bulbar weakness commonly develops, and respiratory weakness
requiring ventilatory support
3. In most patients, weakness progresses for 13 weeks, but rapid
deterioration to respiratory failure can develop within hours.
4. On examination there is diffuse weakness with widespread loss of reflexes.
5. An unusual axonal variant described by Miller Fisher comprises the triad of
ophthalmoplegia, ataxia and areflexia.
6. Overall, 80% of patients recover completely within 36 months, 4% die,
and the remainder suffer residual neurological disability which can be
severe.
7. Adverse prognostic features include older age, rapid deterioration to
ventilation and evidence of axonal loss on EMG.
Investigations
1. The CSF protein is elevated at some stage of the illness but may be normal
in the first 10 days.

15
2. There is usually no rise in CSF cell number (a lymphocytosis of > 50 106
cells/L suggests an alternative diagnosis). Albumino cytologic dissociation!
3. Electrophysiological studies are often normal in the early stages but show
conduction block and multifocal motor slowing, delayed F-waves.
4. Investigation to identify an underlying cause, such as cytomegalovirus,
mycoplasma or Campylobacter, requires a chest X-ray, stool culture and
appropriate immunological blood tests.
5. Antibodies to the ganglioside GQ1b are found in the Miller Fisher variant.
6. Acute porphyria should be excluded by urinary porphyrin estimation, and
serum lead should be measured if there are only motor signs.
Management
1. Regular monitoring of respiratory function (vital capacity and arterial blood
gases) is required, as respiratory failure may develop with little warning
and require ventilatory support.
2. Ventilation may be needed if the vital capacity falls below 1 L, but
intubation is more often required because of bulbar incompetence leading
to aspiration.
3. General management to protect the airway and prevent pressure sores
and venous thrombosis is essential.
4. Corticosteroid therapy has been shown by RCT to be ineffective.
5. plasma exchange and intravenous immunoglobulin therapy shorten the
duration of ventilation and improve prognosis, provided treatment is
started within 14 days of the onset of symptoms

MYASTHENIA GRAVIS; ADD A NOTE ON LEMS - Paper II AUG 2007


1. This condition is characterised by progressive fatigable weakness,
particularly of the ocular, neck, facial and bulbar muscles.
Pathophysiology:
1. The disease is caused by:
1. Autoantibodies to acetylcholine receptors which reduce the number of
acetylcholine receptors and damages the end plate.
2. Autoantibodies to a muscle-specific kinase (MuSK), involved in the
regulation and maintenance of the acetylcholine receptors.
3. Thymoma,
4. Thymic follicular hyperplasia.
5. There is an increased incidence of other autoimmune diseases, and the
disease is linked with certain HLA haplotypes.
2. Drug induced:
a. Penicillamine can cause an antibody-mediated myasthenic
syndrome
b. Aminoglycosides and ciprofloxacin, may exacerbate the
neuromuscular blockade
Clinical features
1. The disease usually presents between the ages of 15 and 50 years.
2. Women are affected more often than men in the younger age groups while
the reverse applies to older age groups.
3. It tends to run a relapsing and remitting course, especially during the early
years.
4. The cardinal symptom is fatigable weakness of the muscles; movement is
initially strong but rapidly weakens as muscle use continues.

16
5. Worsening of symptoms towards the end of the day or following exercise is
characteristic.
6. There are no sensory signs or signs of involvement of the CNS
7. Weakness of the oculomotor muscles may mimic a central eye movement
disorder. The first symptoms are usually intermittent ptosis or diplopia, but
weakness of chewing, swallowing, speaking or limb movement also occurs.
8. The patient is unable to undertake tasks above shoulder level such as
combing the hair without frequent rests.
9. Respiratory muscles may be involved, and respiratory failure is a common
cause of death.
10.Aspiration may occur if the cough is ineffectual.
11.Sudden weakness from a cholinergic or myasthenic crisis may require
ventilatory support.
The prognosis:
1. It is variable and sometimes remissions occur spontaneously.
2. When myasthenia is confined to the eye muscles, the prognosis is
excellent and disability slight.
3. Young female patients with generalised disease have high remission rates
after thymectomy, whilst older patients are less likely to have a remission
despite treatment.
4. Rapid progression of the disease more than 5 years after its onset is
uncommon.
Investigations
1. Tensilon test: 2 mg is injected initially, with a further 8 mg given half a
minute later if there are no undesirable side-effects. Improvement in
muscle power occurs within 30 seconds and usually persists for 23
minutes.
2. EMG with repetitive stimulation may show the characteristic decremental
respons.
3. Antiacetylcholine receptor antibody (AChRA) is found in over 80% of cases
4. Anti-MuSK antibodies are found in patients with prominent bulbar
involvement.
5. Positive anti-skeletal muscle antibodies suggest the presence of thymoma,
but all patients should have a thoracic CT to exclude this condition which
may not be visible on plain X-ray examination.
6. Screening for associated autoimmune disorders, particularly thyroid
disease, is important.
Management
The principles of treatment are:
1. To maximise the activity of acetylcholine at remaining receptors in the
neuromuscular junctions
2. To limit or abolish the immunological attack on motor end plates.
3. Pyridostigmine is given orally in a dosage of 30120 mg, usually 6-hourly.
Muscarinic side-effects, including diarrhoea and colic, may be controlled by
propantheline (15 mg as required).
4. The immunological treatment of myasthenia:
a. Thymectomy: considered in any antibody-positive patient under 45
years
b. Plasma exchange: reserved for myasthenic crisis or for preoperative preparation

17
c. Intravenous immunoglobulin: for short-term treatment of severe
myasthenia
d. Corticosteroid treatment: Improvement is commonly preceded
by marked exacerbation of myasthenic symptoms and treatment
should be initiated in hospital
e. Other immunosuppressant treatment: Treatment with
azathioprine 2.5 mg/kg daily is of value in reducing the dosage of
steroids necessary and may allow steroids to be withdrawn
5. Thymectomy in the early stages of the disease improves overall prognosis.
Other myasthenic syndromes: LambertEaton myasthenic syndrome
(LEMS)
1. LambertEaton myasthenic syndrome (LEMS), is a myasthenic syndrome in
which transmitter release is impaired, often in association with antibodies
to prejunctional voltage-gated calcium channels.
2. Patients may have autonomic dysfunction (and a dry mouth) in addition to
muscle weakness, but the cardinal clinical sign is absence of tendon
reflexes, which can return immediately after sustained contraction of the
relevant muscle.
3. The condition is associated with underlying malignancy in a high
percentage of cases
4. The condition is diagnosed electrophysiologically by the presence of posttetanic potentiation of motor response to nerve stimulation at a frequency
of 2050/s.
5. Treatment is with 3,4-diaminopyridine.

MULTIPLE SCLEROSIS
Definition:
Multiple sclerosis, an idiopathic inflammatory disease of the central
nervous system, is characterized pathologically by demyelination and
subsequent axonal degeneration.
Incidence:
The annual incidence of around 7 per 100 000. The lifetime risk of
developing MS is about 1 in 400.
The disease commonly presents in young adults and affects twice as many
women as men.
Pathophysiology
1. The prevalence has been found to correlate with various environmental
factors, such as sunlight exposure, vitamin D and exposure to
EpsteinBarr virus (EBV)
2. Genetic factors: Monozygotic twins have a concordance rate of 30%.
inheritance appears to be polygenic, with influences from the HLA region
3. An immune mechanism is suggested by increased levels of activated T
lymphocytes in the CSF and increased immunoglobulin synthesis within the
CNS.
Pathology
1. CNS inflammation starts with the entry of activated T lymphocytes across
the bloodbrain barrier. The resulting inflammatory cascade releases

18
cytokines and initiates destruction of the oligodendrocytemyelin unit by
macrophages.
2. Histologically, inflammatory demyelination occurring most commonly in
the periventricular regions of the brain, the optic nerves and the subpial
regions of the spinal cord.
3. After an acute attack, gliosis follows, leaving a shrunken grey scar.
4. In established MS there is progressive axonal loss, with progressive and
persistent disability.
Clinical features
1. A diagnosis of MS requires the demonstration of lesions in more than one
anatomical site at more than one time. This is described as dissemination
in time and space.
2. Patients have a relapsing and remitting clinical course of episodic
dysfunction of the CNS with variable recovery. Some have a slowly
progressive clinical course, and a tiny minority who have a fulminant
variety leading to early death.
3. The peak age of onset is in the fourth decade and onset before puberty or
after the age of 60 years is rare.
4. Common presentations of multiple sclerosis
a. Optic neuritis
b. Relapsing and remitting sensory symptoms
c. Subacute painless spinal cord lesion
d. Acute brain-stem syndrome
e. Subacute loss of function of upper limb (dorsal column deficit)
f. 6th cranial nerve palsy
5. Other symptoms and syndromes suggestive of CNS demyelination
a. Afferent pupillary defect and optic atrophy (previous optic neuritis)
b. Lhermittes symptom (tingling in spine or limbs on neck flexion)
c. Progressive non-compressive paraparesis
d. Partial BrownSquard syndrome
e. Internuclear ophthalmoplegia with ataxia
f. Postural (rubral, Holmes) tremor
g. Trigeminal neuralgia under the age of 50
h. Recurrent facial palsy
Investigations
a. There is no specific test for MS .
b. Visual evoked potentials can detect clinically silent lesions in up to 70% of
patients
c. The CSF may show a lymphocytic pleocytosis in the acute phase and
oligoclonal bands of IgG
d. MRI appearances in MS may be those of cerebrovascular disease or
cerebral vasculitis.
Diagnosis

a.

The Macdonald criteria for the diagnosis of multiple sclerosis


a. Two or more attacks separated in time (at least 3 months apart)
and space (involving different parts of the CNS) with clinical
evidence of two or more lesions
a. If there is objective clinical evidence for only one lesion:
a. Dissemination in space demonstrated by MRI
b. Two or more MRI-detected lesions consistent with MS
c. oligoclonal bands in CSF or
d. Await further clinical attack at different anatomical site

19
Management
The acute episode
a. Pulses of high-dose methylprednisolone, either intravenously (1 g daily for
3 days) or orally (500 mg daily for 5 days), shorten the duration of the
episode.
b. Pulses of steroids can be given up to three times in a year but their
administration should be restricted to those with significant functionthreatening deficits.
c. Prophylaxis to prevent the occurrence of steroid-induced osteoporosis
should be considered
Prevention of relapse
a. Immunosuppressive agents such as azathioprine appear to reduce the risk
of relapses and improve longterm outcome.
b. subcutaneous or intramuscular interferon beta reduces the number of
relapses
c. The recently introduced agent, natalizumab, is reserved for patients with
particularly aggressive disease, along with less proven therapies such as
mitoxantrone and cyclophosphamide.
d. Special diets, including a gluten-free diet or linoleic acid supplements, and
hyperbaric oxygen therapy are popular with patients but are of no proven
benefit.
Treatment of complications and disability
Complication
Treatment
Spasticity
Physiotherapy; Baclofen 15100 mg* (oral)
(in divided doses) Dantrolene 25100
mg*(in divided doses)
Local (i.m.) injection of botulinum toxin
Chemical neuronectomy
Ataxia
Isoniazid 6001200 mg (in divided
doses)
Clonazepam 28 mg (in divided doses)
Dysaesthesia
Carbamazepine 2001800 mg (in
divided doses)
Gabapentin 9002400 mg (in divided
doses)
Phenytoin 200400 mg
Amitriptyline 10100 mg
Bladder symptoms
Intermittent urinary catheterisation
Fatigue
Amantadine 100300 mg

DISCUSS AETIOLOGY, CLINICAL FEATURES, INVESTIGATIONS AND


MANAGEMENT OF PARAPLEGIA.NOV 2001
DEFINITION: Paraplegia is impairment in motor and or sensory functions of the
lower extremities often including lower part of the trunk.
Paraplegia; total paralysis

20
Paraperesis: partial paralysis
Spinal cord levels: thoracic; lumbar; sacral
Most
1.
2.
3.

common causes of Paraplegia in Ts


Trauma
Tumor
Tuberculosis

4. Transverse myelitis
Classification of causes:
1. Traumatic
2. Non Traumatic:
a. Compresssive:
i. Extradural: Potts disease, Metastatic carcinoma, Multiple
myeloma, Cervical spondylosis, Epidural abscess,
Echinococcus cyst
ii. Intradural:
1. Extramedulaary: Neurofibroma, Meningioma
2. Intramedullary: Astrocytoma, Ependymoma,
Tuberculoma, Schistosome ova, Syringomyelia
b. Non compressive:
i. Transverse myelopathy: viral infections, HIV, TB, syphilis,
HTLV-1, Devics disease
ii. Nutritional Toxins: konzo, lathyrism, tropical ataxic
neuropathy
iii. Nutritional deficiency: B-12
iv. Vascular: sickle cell disease, dural AV fistula
v. Hereditary: familial spastic paraplegia
Evolution of paraplegia:
1. Vascular lesions: Sudden onset (over minutes) followed by a stable
period and gradual recovery
2. Neoplastic lesions: Gradual in onset and progressive over weeks or
months

21
3. Inflammatory lesions: Fairly acute in onset (over a few days), persist for
a time and then improve (e.g. in multiple sclerosis)
4. Degenerative disorders: Evolve over months or years (e.g. motor
neuron disease or cervical spondylotic myelopathy)

Spinal cord compression:


Clinical features:

1. Pain: Localised over the spine or in a root distribution, which may be


aggravated by coughing, sneezing or straining
2. Sensory: Paraesthesia, numbness or cold sensations, especially in the
lower limbs, which spread proximally, often to a level on the trunk
3. Motor: Weakness, heaviness or stiffness of the limbs, most commonly the
legs
4. Sphincters: Urgency or hesitancy of micturition, leading eventually to
urinary retention
Signs of spinal cord compression
1. Cervical, above C5:
a. Upper motor neuron signs and sensory loss in all four limbs
b. Diaphragm weakness (phrenic nerve)
2. Cervical, C5T1:
a. Lower motor neuron signs and segmental sensory loss in the arms;
upper motor neuron signs in the legs
b. Respiratory (intercostal) muscle weakness
3. Thoracic cord: Spastic paraplegia with a sensory level on the trunk
4. Conus medullaris: Weakness of legs, sacral loss of sensation and
extensor plantar responses
5. Cauda equine: Spinal cord ends at approximately the T12/L1 spinal level
and spinal lesions below this level can only cause lower motor neuron
signs by affecting the cauda equine
Investigations
1. CT, MRI and myelography can localise the lesion and can define the extent
of compression and associated soft-tissue abnormality.
2. Plain X-rays may show bony destruction and soft-tissue abnormalities
3. Routine investigations, including chest X-ray, may provide evidence of
systemic disease like TB
4. If myelography is performed, CSF should be taken for analysis; in cases of
complete spinal block this shows a normal cell count with a very elevated
protein causing yellow discoloration of the fluid (Froins syndrome).
5. In patients with secondary tumours, needle biopsy may be required to
establish a tissue diagnosis, unless this has already been established.
Management
1. Benign tumours should be surgically excised
2. Extradural compression due to malignancy:
a. Poor prognosis,
b. Surgical decompression
c. Radiotherapy.

22
3. Spinal cord compression due to tuberculosis: surgical treatment if seen
early. This should be followed by appropriate anti-tuberculous
chemotherapy for an extended period.
4. Traumatic lesions of the vertebral column require specialised neurosurgical
treatment.

Clinical features of Non compressive (intrinsic) myelopathy:


1. Congenital:
a. Diastematomyelia (spina bifida)
i. Features variably present at birth and deteriorate thereafter
LMN features, deformity and sensory loss of legs
ii. Impaired sphincter function
iii. Hairy patch or pit over low back
iv. Incidence reduced by increased maternal intake of folic acid
during pregnancy
b. Hereditary spastic paraplegia
i. Onset usually in adult life
ii. Autosomal dominant inheritance usual
iii. Slowly progressive UMN features affecting legs > arms
iv. Little, if any, sensory loss
2. Infective/inflammatory:
a. Transverse myelitis due to viruses (HZV), schistosomiasis,HIV, MS,
sarcoidosis:
i. Weakness and sensory loss, often with pain, developing over
hours to days
ii. UMN features below lesion Impaired sphincter function
b. Vascular
i. Anterior spinal artery infarct: Abrupt onset
ii. Intervertebral disc embolus:
1. Anterior horn cell loss (LMN) at level of lesion
2. UMN features below it
3. Spinothalamic sensory loss below lesion but spared
dorsal column sensation
c. Neoplastic: Glioma, ependymoma
i. Weakness and sensory loss, often with pain, developing over
months to years
ii. UMN features below lesion in cord; additional LMN features in
conus
iii. Impaired sphincter function
d. Metabolic Vitamin B12 deficiency (subacute combined
degeneration)
i. Progressive spastic paraparesis with proprioception loss,
absent reflexes due to peripheral neuropathy optic nerve
and cerebral involvement
e. Degenerative:
i. Motor neuron disease
1. Relentlessly progressive LMN and UMN features,
associated bulbar weakness, no sensory involvement
ii. Syringomyelia
1. Gradual onset over months or years, pain in cervical
segments

23
2. Anterior horn cell loss (LMN) at level of lesion, UMN
features below it
3. Suspended spinothalamic sensory loss at level of
lesion, dorsal columns preserved.
Investigation of intrinsic disease:
1. Starts with imaging to exclude a compressive lesion. If available,
2. MRI provides the most information about structural lesions such as
diastematomyelia, syringomyelia or intrinsic tumours.
3. Non-specific signal change may be seen in the spinal cord in inflammatory
or infective conditions and others such as B12 deficiency.
4. Other investigations such as lumbar puncture or blood tests are required
to make a specific diagnosis
Management:
1. Transverse myelitis:
a. Treatment initially is with high dose intravenous steroids in
combination with a course of antiviral medication e.g. acyclovir 10
mg/kg (800 mg) iv/po/3-4 times daily for 10 days if clinically
appropriate.
b. High dose steroids include methylprednisolone 1000 mg iv daily for
5 days followed by oral prednisolone 60 mg daily, tapering over 2-3
weeks. High dose dexamethasone, 24-32 mg daily can also be used
if methylprednisolone is unavailable.
2. B12 deficiency:
a. Treatment is with intramuscular injections of B-12. The dose is 1 mg
on alternate days for a total of five injections followed by 1 mg
injections every 3 months for life.
b. If intramuscular injections are not available then B-12 can
sometimes be given orally, in a dose of 1 mg po daily.
Supportive care:
Bladder and bowels
1. Urinary catheterization is necessary when there is a non functioning
bladder.
2. It may be necessary to use antibiotics for urinary tract infections.
3. Constipation: satisfactory fluid intake, adequate fibre/bulk in the diet, and
the early use of laxatives. Manual evacuation may be necessary when
there is faecal impaction.
Limbs
1. Care of paralysed limbs involves frequent passive movements. This helps
to prevent thrombosis, joint stiffness, spasticity and contractures, and to
exercise the non affected muscles.
2. The main antispasmodics available are baclofen and diazepam. The
starting dose of baclofen is 5 mg twice daily increasing slowly over weeks
to 20-40 mg twice daily as required.
3. Other drugs used for spasticity include dantrolene and tizanidine. Further
measures include the use of support stockings and low dose heparin which
help to prevent deep vein thrombosis.

24

Skin: protective cushions to guard against pressure points. Regular


inspection of skin is essential and may of necessity have to be carried out
by the carer.

ENUMERATE THE CLINICAL TYPES OF CNS TUBERCULOSIS. DISCUSS


THE INVESTIGATIONS AND MANAGEMENT OF TBM OCT 2003
Central nervous system tuberculosis includes:
1. Tubercular Meningitis(TBM)
2. Parenchymal Tuberculomas
3. Miliary Tuberculomas
4. Tubercular Encephalopathy ( more common in children)
5. Tubercular Hypophysitis (Pituitary)
6. Spinal Tuberculosis
a. Tubercular Spondylitis (Potts spine)
b. Non-osseous spinal tuberculomas
c. Tubercular arachnoiditis (myeloradiculopathy)
d. Spinal meningitis
e. Tubercular myelitis
Tuberculous meningitis
Pathophysiology
1. Tuberculous meningitis occurs most commonly shortly after a primary
infection in childhood or as part of military tuberculosis.
2. The usual local source of infection is a caseous focus in the meninges or
brain substance adjacent to the CSF pathway.
3. The brain is covered by a greenish, gelatinous exudate, especially around
the base, and numerous scattered tubercles are found on the meninges.
Clinical features
1. Symptoms
1. Headache
2. Vomiting
3. Low-grade fever
4. Lassitude
5. Depression
6. Confusion
7. Behaviour changes
2. Signs
1. Meningism (may be absent)
2. Oculomotor palsies
3. Papilloedema
4. Depression of conscious level
5. Focal hemisphere signs
3. Untreated tuberculous meningitis is fatal in a few weeks but complete
recovery is the rule if treatment is started before the appearance of focal
signs or stupor. When treatment is started at a later stage, the recovery
rate is 60% or less and the survivors show permanent neurological deficit.
Investigations
1. Lumbar puncture should be performed if the diagnosis is suspected.
2. Findings:
a. The CSF is under increased pressure.

25
b. It is usually clear but, when allowed to stand, a fine clot (spider
web) may form.
c. The fluid contains up to 500 106 cells/L, predominantly
lymphocytes.
d. There is a rise in protein and a marked fall in glucose.
e. The tubercle bacillus may be detected in a smear of the centrifuged
deposit from the CSF but a negative result does not exclude the
diagnosis.
f. The CSF should be cultured but, as this result will not be known for
up to 6 weeks, treatment must be started without waiting for
confirmation.
3. Brain imaging may show hydrocephalus, brisk meningeal enhancement on
enhanced CT and/or an intracranial tuberculoma.
4. CSF Nucleic acid assay (NAA) particularly polymerase chain reaction (PCR)
assay, has emerged as a promising new method
Management
1. As soon as the diagnosis is made or strongly suspected, chemotherapy
should be started using one of the regimens including pyrazinamide
described on page 693.
2. First-line: 4 drugs for 2 months, then isoniazid plus rifampicin for 10
monthsa
Isoniazid
300 mg
Rifampicin
450 mg (<50 kg) or 600 mg (50 kg)
Pyrazinamide
1.5 g (<50 kg) or 2.0 g (50 kg)
Ethambutol or
Ethambutol: 15 mg/kg (maximum, 1 g); streptomycin:
d
streptomycin
15 mg/kg
3. The use of corticosteroids in addition to antituberculous therapy has been
controversial. Recent evidence suggests that it improves mortality but not
focal neurological damage, especially if given early.
a. Dexamethasone IV/PO 0.4 mg/kg
initially, then reduce course over 68
weeks.
b. Prednisolone

1-2 mg/kg (PO or


intravenous) for 4 weeks,
then reduce course over
next 4 weeks
4. Surgical ventricular drainage may be needed if obstructive hydrocephalus
develops.
5. Skilled nursing is essential during the acute phase of the illness, and
measures should be put in place to maintain adequate hydration and
nutrition.

MENTION THE CAUSES AND MANAGEMENT OF SUBARTACHNOID


HEMOORHAGE. DEC 91
1. Subarachnoid haemorrhage (SAH) is less common than other types of
stroke and affects women more commonly than men and the condition
usually presents before the age of 65.
2. The immediate mortality of aneurysmal subarachnoid haemorrhage is
about 30% and survivors have a recurrence, or rebleed, rate of about 40%
in the first 4 weeks and 3% annually thereafter.
Etiology

26
1. Eighty-five percent of SAH are caused by saccular or berry aneurysms
arising from the bifurcation of cerebral arteries, particularly in the region of
the circle of Willis.
2. There is an increased risk in:
a. First-degree relatives of those with saccular aneurysms,
b. In patients with polycystic kidney disease
c. Congenital connective tissue defects such as EhlersDanlos
syndrome.
3. In about 10% of cases, SAH are non-aneurysmal haemorrhages
4. Some 5% of SAH are due to arteriovenous malformations and vertebral
artery dissection.
Clinical features
1. presents with a sudden, severe thunderclap headache accompanied by
vomiting.
2. Physical exertion, straining and sexual excitement are common
antecedents.
3. There may be loss of consciousness at the onset
4. patient is distressed and irritable, with photophobia.
5. There may be neck stiffness
6. Focal hemisphere signs such as hemiparesis or aphasia may be present
7. A third nerve palsy may be present due to local pressure from an
aneurysm of the posterior communicating artery, though this is rare.
8. Fundoscopy may reveal a subhyaloid haemorrhage, which represents blood
tracking along the subarachnoid space around the optic nerve.
Investigations
1. Lumbar puncture is the investigation of first choice and should be
performed after 12 hours from symptom onset, if possible.
2. The diagnosis of SAH can be made by CT, but a negative result does not
exclude the diagnosis
3. Cerebral angiography is required to determine the optimal approach to
prevent recurrent bleeding.
Management
1. Insertion of platinum coils into an aneurysm (via an endovascular
procedure) or surgical clipping of the aneurysm
2. Arteriovenous malformations can be managed by surgical removal, ligation
of the blood vessels that feed or drain the lesion, or injection of material to
occlude the fistula or draining veins.
3. Nimodipine (3060 mg i.v. for 514 days, followed by 360 mg orally for a
further 7 days) is usually given to prevent vasospasm in the acute phase.
4. Treatment may also be required for complications of SAH, which include:
a. Obstructive hydrocephalus (possibly requiring drainage via a shunt),
b. Delayed cerebral ischaemia due to vasospasm (vasodilators),
c. Hyponatraemia (water restriction)
d. Systemic complications associated with immobility, such as chest
infection and venous thrombosis.

COMA - OCT 2000

27
1. Persistent loss of consciousness or coma indicates disorder of bilateral
hemisphere or brain-stem disease.
2. The history of the mode of onset is crucial to establishing the cause
3. Assessment and stabilisation of the vital functions is important
4. Neurological examination may reveal important findings, such as:
a. evidence of head injury,
b. papilloedema,
c. meningism or
d. an eye movement disorder.
5. No focal neurological signs are obvious in drug overdose and metabolic
disturbance
Etiologic classification of coma:
1. Metabolic disturbance
a. Drug overdose
b. Diabetes mellitus
i. Hypoglycaemia
ii. Ketoacidosis
iii. Hyperosmolar coma
c. Hyponatraemia
d. Uraemia
e. Hepatic failure
f. Respiratory failure
g. Hypothermia
h. Hypothyroidism
i. Thiamin deficiency
2. Trauma
a. Cerebral contusion
b. Extradural haematoma
c. Subdural haematoma
3. Cerebrovascular disease
a. Subarachnoid haemorrhage
b. Brain-stem infarction/haemorrhage
c. Intracerebral haemorrhage
d. Cerebral venous sinus thrombosis
4. Infections
a. Meningitis
b. Encephalitis
c. Cerebral abscess
d. General sepsis
5. Others
a. Epilepsy
b. Brain tumour
c. Functional (pseudo-coma)
Assessment: Glasco coma scale:
a. Eye-opening (E)
1. Spontaneous 4
2. To speech 3
3. To pain 2
4. Nil 1
b. Best motor response (M)
1. Obeys 6
2. Localises 5

28
3. Withdraws 4
4. Abnormal flexion 3
5. Extensor response 2
6. Nil 1
c. Verbal response (V)
1. Orientated 5
2. Confused conversation 4
3. Inappropriate words 3
4. Incomprehensible sounds 2
5. Nil 1
Coma score = E + M + V Minimum 3; Maximum 15
Evaluation:
1. Mode of onset:
1. Abrupt onset of coma: subarachnoid hemorrhage, brainstem stroke, or
intracerebral hemorrhage
2. Slower onset and progression: structural or mass lesions.
3. Urgent tests: Noncontrast CT scanning: identifes intracranial hemorrhage,
brain herniation, or other structural lesion that may require immediate
neurosurgical intervention.
4. A metabolic cause is likely with a preceding intoxicated state or agitated
delirium. In such case assessment includes:
2. SIGNS OF TRAUMA
1. Inspection of the head may reveal signs of basilar skull fracture,
including the following:
1. Raccoon eyesPeriorbital ecchymoses.
2. Battle's signSwelling and discoloration overlying the mastoid
bone behind the ear.
3. HemotympanumBlood behind the tympanic membrane.
4. Cerebrospinal fluid (CSF) rhinorrhea or otorrhea
2. Palpation of the head may demonstrate a depressed skull fracture or
swelling of soft issues at the site of trauma.
3. Response to Painful Stimuli:
1. Purposive limb withdrawal from painful stimuli implies that sensory
pathways are functionally intact.
2. Unilateral absence of responses despite application of stimuli implies a
corticospinal lesion;
3. bilateral absence of responsiveness suggests brainstem involvement,
bilateral pyramidal tract lesions, or psychogenic unresponsiveness.
4. Decorticate posturing may occur with lesions of the internal capsule
and rostral cerebral peduncle,
4. Ocular Findings
1. Pupils:
a. destructive pontine lesions may lead to small but reactive pupils.
b. Ipsilateral pupillary dilation with no direct or consensual response
to light occurs with compression of the third cranial nerve, eg,
with uncal herniation.
c. The pupils are slightly smaller than normal but responsive to light
in many metabolic encephalopathies; however, they may be
fixed and dilated following overdosage with atropine or
scopolamine, and pinpoint (but responsive) with opioids.
d. Pupillary dilation for several hours following cardiopulmonary
arrest implies a poor prognosis.

29
5. Eye movements;
a. Conjugate deviation of the eyes to the side suggests the presence of
an ipsilateral hemispheric lesion or a contralateral pontine lesion.
b. A mesencephalic lesion leads to downward conjugate deviation.
c. Dysconjugate ocular deviation in coma implies a structural
brainstem lesion
d. Dolls eye manoeuvre: With cortical depression in lightly comatose
patients, a brisk dolls-head eye response is seen. With brainstem
lesions, this oculocephalic reflex becomes impaired or lost,
depending on the site of the lesion.
e. The oculovestibular reflex is tested by caloric stimulation using
irrigation with ice water. In normal subjects, jerk nystagmus is
elicited for about 2 or 3 minutes, with the slow component toward
the irrigated ear. In unconscious patients with an intact brainstem,
the fast component of the nystagmus disappears, so that the eyes
tonically deviate toward the irrigated side for 23 minutes before
returning to their original position. With impairment of brainstem
function, the response becomes perverted and finally disappears. In
metabolic coma, oculocephalic and oculovestibular reflex responses
are preserved, at least initially.
6. Respiratory Patterns
a. Cheyne-Stokes respiration (in which episodes of deep breathing
alternate with periods of apnea) may occur with bihemispheric or
diencephalic disease or in metabolic disorders.
b. Hyperventilation occurs with lesions of the brainstem tegmentum;
7. Clinical picture in Structural Lesions:
a. Supratentorial mass lesions tend to affect brain function in an
orderly way. The patient becomes drowsy, then stuporous, and
finally comatose.
b. In contrast, a subtentorial (ie, brainstem) lesion may lead to an
early, sometimes abrupt disturbance of consciousness without any
orderly progression of neurologic signs.
8. Coma Due to Metabolic Disturbances
a. Patients with a metabolic cause of coma generally have neurologic
involvement that cannot be explained by any level in CNS
b. pupillary reactivity is usually preserved
9. Comatose patients with meningitis, encephalitis, or subarachnoid hemorrhage
may exhibit clinical evidence of meningeal irritation: Examination of the
cerebrospinal fluid in such patients is essential to establish the correct
diagnosis.
Investigations:
1. At presentation blood will be taken for determination of glucose,
electrolytes, liver function, calcium, osmolality, and blood gases. Blood
should also be stored for a subsequent drug screen if needed.
2. CT scanning is of greater urgency when a structural lesion is suspected.
3. In the absence of focal signs, but with evidence of meningitis, a lumbar
puncture may need to be performed before scanning, as a matter of
clinical urgency. In other situations, lumbar puncture should be delayed
until after the brain scan because of the risk of precipitating a pressure
cone secondary to a cerebral mass lesion.

30
4. All patients will require chest radiography and ECG, and more detailed
investigations of systemic disease will be directed by the clinical
examination.
5. The EEG can be of value in identifying the status epilepticus. An isoelectric
EEG may occur with drug-induced comas, but otherwise indicates severe
cerebral damage.
Management of the unconscious patient
Emergency Measures:
1. The diagnostic workup proceeds concomitantly with management.
2. Supportive therapy for respiration or blood pressure is initiated; in
hypothermia, patients should be rewarmed before the prognosis is
assessed.
3. The patient can be positioned on one side with the neck partly extended,
dentures removed, and secretions cleared by suction; if necessary, the
patency of the airways is maintained with an oropharyngeal airway.
4. Blood is drawn for serum glucose, electrolyte, and calcium levels; arterial
blood gases; liver and kidney function tests; and toxicologic studies as
indicated.
5. Dextrose 50% (25 g), naloxone (0.41.2 mg), and thiamine (100 mg) are
given intravenously without delay.
6. The management of the unconscious patient will consist of treatment of
the underlying cause where possible, and the maintenance of the normal
physiology in terms of respiration, circulation, and nutrition while the
patient is unconscious.
7. The unconscious patient should be nursed on his or her side without a
pillow, and attention will clearly need to be paid to the airway, requiring an
oral airway as a minimum, although usually patients will require intubation
and, if coma is prolonged, tracheostomy.
8. The unconscious patient will have retention or incontinence of urine and
will require catheterization.
9. Intravenous fluid is necessary and, if coma persists, adequate nutrition is
required. Disturbances of electrolytes, particularly sodium, are common in
the intensive care situation and need scrupulous monitoring.
BRAIN DEATH
1. Patents with prolonged coma as the result of severe and irreversible brain
damage may nowadays survive for prolonged periods with supportive care
and mechanical ventilation. In this situation diagnostic criteria for brain
death have been established in order that patients without functioning
brains who have no chance of recovery may be identified and have their
ventilation discontinued.
2. Preconditions for considering a diagnosis of brain death
a. The patient is deeply comatose
b. There must be no suspicion that coma is due to depressant drugs,
such as narcotics, hypnotics, tranquillisers
c. Hypothermia has been excludedrectal temperature must exceed
35 C
d. There is no profound abnormality of serum electrolytes, acidbase
balance or blood glucose concentrations, and any metabolic or
endocrine cause of coma has been excluded

31
e. The patient is maintained on a ventilator because spontaneous
respiration had been inadequate or had ceased.
f. Drugs, including neuromuscular blocking agents, must have been
excluded as a cause of the respiratory failure
g. The diagnosis of the disorder leading to brain death has been firmly
established. There must be no doubt that the patient is suffering
from irremediable structural brain damage
3. Tests for confirming brain death
a. All brain-stem reflexes are absent
b. The pupils are fixed and unreactive to light
c. The corneal reflexes are absent
d. The vestibulo-ocular reflexes are absentthere is no eye movement
following the injection of 20 mL of ice-cold water into each external
auditory meatus in turn
e. There are no motor responses to adequate stimulation within the
cranial nerve distribution
f. There is no gag reflex and no reflex response to a suction catheter in
the trachea
g. No respiratory movement occurs when the patient is disconnected
from the ventilator long enough to allow the carbon dioxide tension
to rise above the threshold for stimulating respiration (PaCO2 must
reach 6.7 kPa (50 mmHg))

MENINGOCOCCAL MENINGITIS. DEC 91


Meningococcal meningitis (Neisseria meningitidis)
Etiology:
a. The organism is an aerobic Gram negative diplococcus with a
polysaccharide capsule.
b. The meningococcus infects humans only, and it colonizes the nasopharynx.
c. Spread is by aerosol inhalation and close personal contact.
d. The incubation period is short, less than 10 days and the clinical
declaration of meningitis may be explosive.
Clinical features:
a. High fever,chills, and headache; back, abdominal, and extremity pains; and
nausea and vomiting are typical.
b. Rapidly developing confusion, delirium, seizures, and coma occur in some.
On examination, nuchal and back rigidity are typical.
c. Positive Kernig and Brudzinski signs (Kernig sign is pain in the hamstrings
upon extension of the knee with the hip at 90-degree flexion; Brudzinski
sign is flexion of the knee in response to flexion of the neck) are pecific but
not sensitive findings.
d. A petechial rash appearing in the lower extremities and at pressure points
is found in most cases. Petechiae may vary in size from pinpoint lesions to
large ecchymoses or even skin gangrene that may later slough if the
patient survives - purpura fulminans
e. If there is fulminant meningococcaemia, disseminated intravascular
coagulation is triggered and ecchymoses appear which may progress to
gangrene with resultant spontaneous amputation.

32
f.

Adrenal haemorrhage may result from severe meningococcal septicaemia


and cause cardiovascular collapse, the WaterhouseFriderichsen
syndrome. In particularly fulminant cases, these findings may be evident
before signs of meningitis are found.
g. Myocarditis, pericarditis, and arthritis may be present and relate to
circulating immune complexes.
Laboratory Findings
a. Lumbar puncture typically reveals a cloudy or purulent cerebrospinal fluid,
with elevated pressure, increased protein, and decreased glucose content.
The fluid usually contains > 1000 cells/mcL, with polymorphonuclear cells
predominating and containing gram-negative intracellular diplococci.
b. The capsular polysaccharide can be demonstrated in cerebrospinal fluid or
urine by latex agglutination;
c. The organism is usually demonstrated by smear and culture of the
cerebrospinal fluid, oropharynx, blood, or aspirated petechiae.
Differential Diagnosis
a. Meningococcal meningitis must be differentiated from other meningitides.
b. Rickettsial, echovirus and, rarely, other bacterial infections (eg,
staphylococcal infections, scarlet fever) also cause petechial rash.
Prevention
a. Two vaccines, meningococcal polysaccharide vaccine (MPSV4, indicated for
vaccination of persons over age 55) and a conjugate vaccine (MCV4,
indicated for persons aged 255 years) are effective for meningococcal
groups.
b. The Advisory Committee on Immunization Practices recommends
immunization with a dose of MCV4 for preadolescents ages 1112 with a
booster at age 16.
c. Vaccine is also recommend as a two-dose primary series administered 2
months apart for persons aged 2 through 54 years with persistent
complement deficiency, persons with functional or anatomic asplenia, and
for adolescents with HIV infection.
d. All other persons at increased risk for meningococcal disease (eg, military
recruits, microbiologists, or travelers to an epidemic or highly endemic
country) should receive a single dose.
e. Eliminating nasopharyngeal carriage: Rifampin, 600 mg orally twice a day
for 2 days, ciprofloxacin, 500 mg orally once, or one intramuscular 250-mg
dose of ceftriaxone is effective.
Treatment
a. Blood cultures must be obtained and intravenous antimicrobial therapy
started immediately.
b. Aqueous penicillin G is the antibiotic of choice (24 million units/ 24 h
intravenously in divided doses every 4 hours).
c. Penicillin-intermediate strains thus far remain fully susceptible to
ceftriaxone and other third-generation cephalosporins used to treat
meningitis, and these should be effective alternatives to penicillin.
d. In penicillin-allergic patients or those in whom Haemophilus influenzae or
gram-negative meningitis is a consideration, ceftriaxone, 2 g intravenously
every 12 hours, should be used. Treatment should be continued in full
doses by the intravenous route until the patient is afebrile for 5 days.
Shorter coursesas few as 4 days if ceftriaxone is usedare also effective.

33

TRANSIENT ISCHEMIC ATTACK:


1. Definition: Transient ischemic attacks are characterized by focal ischemic
cerebral neurologic deficits that last for < 24 hours (usually < 12 hours).
2. About 30% of patients with stroke have a history of transient ischemic attacks,
and proper treatment of the attacks is an important means of prevention.
3. The incidence of stroke is increased in patients with hypertension or diabetes
mellitus.
4. The risk of stroke is highest in the month after a transient ischemic attack
(particularly in the first 48 hours) and progressively declines thereafter.
Etiology
1. An important cause of transient cerebral ischemia is embolization.
2. Cardiac causes of embolic ischemic attacks include:
a. Atrial fibrillation,
b. Rheumatic heart disease,
c. Mitral valve disease,
d. Infective endocarditis,
e. Atrial myxoma, and
f. Mural thrombi complicating myocardial infarction.
g. Atrial septal defects and patent foramen ovale may permit emboli from
the veins to reach the brain (paradoxical emboli).
h. An ulcerated plaque on a major artery to the brain may serve as a
source of emboli.
i. In the anterior circulation, atherosclerotic changes occur most
commonly in the region of the carotid bifurcation extracranially, and
these changes may cause a bruit.
j. Patients with AIDS have an increased risk of developing transient
ischemic deficits or strokes.
k. Less common abnormalities of blood vessels that may cause transient
ischemic attacks include:
i. Fibromuscular dysplasia, which affects particularly the cervical
internal carotid artery;
ii. Atherosclerosis of the aortic arch;
iii. Inflammatory arterial disorders such as giant cell arteritis,
iv. Systemic lupus erythematosus,
v. Polyarteritis, and granulomatous angiitis; and
vi. Meningovascular syphilis.
vii. Hypotension may cause a reduction of cerebral blood flow if a
major extracranial artery to the brain is markedly stenosed, but
this is a rare cause of transient ischemic attack.
l. Hematologic causes of ischemic attacks include:
i. Polycythemia,
ii. Sickle cell disease, and
iii. Hyperviscosity syndromes.
iv. Severe anemia may also lead to transient focal neurologic
deficits in patients with preexisting cerebral arterial disease.
m. The subclavian steal syndrome may lead to transient vertebrobasilar
ischemia. Symptoms develop when there is localized stenosis or
occlusion of one subclavian artery proximal to the source of the
vertebral artery, so that blood is stolen from this artery. A bruit in the
supraclavicular fossa, unequal radial pulses, and a difference of 20 mm

34
Hg or more between the systolic blood pressures in the arms should
suggest the diagnosis in patients with vertebrobasilar transient
ischemic attacks.
Clinical Findings
A. Symptoms and Signs
1. Onset is abrupt and without warning, and recovery usually occurs rapidly,
often within a few minutes.
2. Attacks may occur intermittently over a long period of time, or they may stop
spontaneously.
3. In general, carotid ischemic attacks are more liable than vertebrobasilar
ischemic attacks to be followed by stroke.
4. The stroke risk is greater in patients older than 60 years, in diabetics, or after
transient ischemic attacks that last longer than 10 minutes and with
symptoms or signs of weakness, speech impairment, or gait disturbance.
B. Imaging
1. CT or MRI scan: identifies small cerebral hemorrhage or a cerebral tumor
masquerading as a transient ischemic attack and acute or subacute infarction.
2. Carotid duplex ultrasonography is useful for detecting significant stenosis of
the internal carotid artery, and MR or CT angiography permits broader
visualization of cervical and intracranial vasculature.
3. Cerebral arteriography is the gold standard for investigating the integrity of
the cervical and cerebral vasculature, and allows for angioplasty or other
interventions, if necessary.
C. Laboratory and Other Studies
1. include assessment for hypertension, heart disease, ematologic disorders,
diabetes mellitus, hyperlipidemia, and peripheral vascular disease.
2. It should include complete blood count, fasting blood glucose and serum
cholesterol and homocysteine determinations, serologic tests for syphilis, and
an ECG and chest radiograph.
3. Echocardiography with bubble contrast is performed if a cardiac source is
likely, and blood cultures are obtained if endocarditis is suspected.
4. Holter monitoring is indicated if a transient, paroxysmal disturbance of cardiac
rhythm is suspected.
Differential Diagnosis
1. Focal seizures usually cause abnormal motor or sensory phenomena such as
clonic limb movements, paresthesias, or tingling, rather than weakness or loss
of feeling. Symptoms generally spread (march) up the limb and may lead to
a generalized tonic-clonic seizure.
2. Classic migraine is easily recognized by the visual premonitory symptoms,
followed by nausea, headache, and photophobia, but less typical cases may
be hard to distinguish. The patients age and medical history (including family
history) may be helpful in this regard. Patients with migraine commonly have
a history of episodes since adolescence and report that other family members
have a similar disorder.
3. Focal neurologic deficits may occur during periods of hypoglycemia in diabetic
patients receiving insulin or oral hypoglycemic agent therapy.
Treatment
1. Hospitalization should be considered for patients seen within 72 hours of the
attack, when they are at increased risk for early recurrence.

35
2. One commonly used method to assess recurrence risk is the ABCD2 score;
points are assigned for each of the following criteria:
a. Age 60 years or older (1 point),
b. Blood pressure 140/90 mm hg (1 point),
c. Clinical symptoms of focal weakness (2 points) or
d. Speech impairment without weakness (1 point),
e. Duration 60 minutes (2 points) or 1059 minutes (1 point), or
f. Diabetes mellitus (1 point).
3. An ABCD2 score of 3 or more points has been suggested as a threshold for
hospital admission.
4. When arteriography reveals stenosis, treatment consist of:
a. carotid endarterectom
b. angioplasty
c. stenting
5. Medical treatment is aimed at preventing further attacks and stroke.
a. Treat diabetes mellitus; hematologic disorders; and hypertension,
preferably with an angiotensin-converting enzyme inhibitor or
angiotensin receptor blocker.
b. Consider starting a statin medication regardless of the current lowdensity lipoprotein level; in addition to reducing stroke risk, antecedent
statin use may improve the outcome if an ischemic stroke does occur.
6. Cigarette smoking should be stopped, and cardiac sources of embolization
should be treated appropriately. Weight reduction and regular physical activity
should be encouraged when appropriate.

ATAXIA: CEREBELLAR DISORDERS


Neuroanatomy
1. The cerebellum can be divided into central structures (lingula, vermis and
flocculonodular lobe) and the cerebellar hemispheres.
2. Main inputs come from frontopontocerebellar connections (contralateral)
from above, and spinocerebellar tracts from below (proprioception)
producing primarily ipsilateral signs.
3. Midline lesions can produce severe gait and truncal ataxia.
4. Cerebellar hemisphere lesions can produce classic ipsilateral limb ataxia
(intention tremor, past pointing and mild hypotonia).
5. Lesions in the frontal lobe can produce "pseudocerebellar signs" because
of the way the frontal lobe influences the cerebellum and basal ganglia
Aetiology
1. Vascular: stroke or transient ischaemic attack (TIA) Usually with other
brainstem features. Infarction of the posterior inferior cerebellar artery
causes lateral medullary syndrome (Wallenberg's syndrome) with
hemiataxia, vertigo, dysarthria, ptosis and miosis
2. Space-occupying: enlarging masses in the cerebellum may obstruct CSF
flow, causing hydrocephalus and raised intracranial pressure.
a. Hydrocephalus
b. Posterior fossa tumours or abscess
c. Coning of the cerebellar tonsils can occur rapidly (within hours),
causing respiratory arrest.
3. Nutritional:

36
a. Thiamine deficiency - Wernicke's encephalopathy (triad of acute
confusion, ataxia and ophthalmoplegia); requires urgent thiamine
treatment
b. Vitamin E deficiency (including a genetic form)
c. Gluten sensitivity (gluten ataxia): Neurological dysfunction can be
the only manifestation of coeliac disease and, in this situation,
typically presents as cerebellar ataxia, peripheral neuropathy
Intestinal symptoms and histological changes may be absent. The
neurological features may reverse with gluten-free diet
d. Zinc deficiency (rarely)
4. Infections:
a. Bacterial: meningo-encephalitis or intracranial abscess
b. Viral: acute infections (eg varicella); chronic infections, eg human
immunodeficiency virus (HIV); post-viral syndromes (eg postinfective
cerebellar syndrome in childhood)
c. Parasitic infections (eg toxoplasma, falciparum malaria, Lyme
disease)
d. Prions: Creutzfeldt-Jakob disease (CJD), kuru
5. Toxins: alcohol, mercury, other heavy metals, solvents, carbon monoxide
poisoning
6. Drugs: barbiturates, phenytoin, piperazine, antineoplastic drugs,
deferiprone[7]
7. Trauma: head injury
8. Degeneration:
a. Multiple sclerosis (MS)
b. Paraneoplastic cerebellar degeneration: This is cerebellar disease
occurring with cancer, but not due to brain secondaries; it may be
related to antibodies It can occur with any cancer but most
commonly with lung, gynaecological or breast cancer and Hodgkin's
lymphoma
9. Genetic: there are a number of inherited cerebellar ataxias: Many of these
present in adulthood. Examples are Friedreich's ataxia (the commonest)
and ataxia telangiectasia
10.Metabolic and endocrine:
a. Cerebral oedema of chronic hypoxia
b. Wilson's disease (rare)
c. Hypothyroidism (rarely)
d. Inherited metabolic disorders, eg Leighs disease and the
mitochondrial disorders
11.Congenital:
a. Developmental anomalies, eg cerebellar hypoplasia, Dandy-Walker
syndrome, Arnold-Chiari malformation
b. Cerebral palsy: There are various other uncommon neurological or
metabolic diseases which may involve the cerebellum
c. Idiopathic cerebellar ataxia - a diagnosis of exclusion
Clinical Presentation:
a. As the cerebellum is associated with motor control, lesions produce a
range of movement disorders (ataxias). These can be differentiated by
their time course.
b. Acute onset ataxia: Either due to cerebellar haemorrhage or infarction.
Haemorrhage presents with:
a. Occipital headache

37
b. Vertigo
c. Vomiting
d. Altered consciousness
c. Subacute ataxia: May occur from:
a. Viral infection - children aged 2-10 years; present with pyrexia, limb
and gait ataxia, dysarthria appearing over hours or days; takes up to
six months for full recovery
b. Post-infectious encephalomyelitis - commonly related to varicella
infection but other organisms may be involved
c. Other causes include - hydrocephalus, posterior fossa tumours,
abscesses, parasitic infections and various toxins
d. Episodic ataxias: This is episodes of ataxia lasting minutes to hours. May
appear bizarre and may be misdiagnosed as being of hysterical origin.
There are various causes:
a. Drugs
b. MS
c. Transient vertebrobasilar ischaemic attacks
d. Foramen magnum compression
e. Inherited periodic ataxia, dysarthria, nystagmus and vertigo
f. Intermittent obstruction of ventricular system of which there are two
types:
i. Brief attacks which may benefit from acetazolamide or
phenytoin and the patient is usually well between attacks.
ii. More prolonged attacks which are often associated with
nausea, vertigo and vomiting. More severe in childhood with
drowsiness, headache and fever and interictal nystagmus;
slow deterioration in the ataxia and responds to
acetazolamide (screen for metabolic disorder).
e. Chronic progressive ataxias:
a. Commonly caused by chronic alcohol abuse associated with
malnutrition May improve with thiamine
b. May also occur with other deficiencies, including zinc and vitamin E
c. Other causes:
i. Ingestion of drugs - especially anticonvulsants, particularly
phenytoin (may reverse once drug stopped)
ii. Heavy metals
iii. Structural lesions
iv. Paraneoplastic cerebellar degeneration associated with
carcinomas of the lung or ovaries
Clinical features:
a. Symptoms and signs of ataxia consist of gait impairment, unclear
("scanning") speech, visual blurring due to nystagmus, hand
incoordination, and tremor with movement.
b. Vertigo that occurs in vestibular nerve or labyrinthine disease is absent in
true cerebellar ataxia
c. Similarly, with sensory ataxia as in dorsal column disease imbalance
dramatically worsens when visual input is removed (Romberg sign). This is
not so in true cerebellar ataxia.
d. Chronic symmetric gait ataxia suggests an inherited ataxia, a metabolic
disorder, or a chronic infection.

38
e. Infectious diseases that can present with ataxia are meningovascular
syphilis and tabes dorsalis due to degeneration of the posterior columns
and spinocerebellar pathways in the spinal cord.
f. Hypothyroidism must always be considered as a readily treatable and
reversible form of gait ataxia.
g. Inherited ataxias:
a. Friedreichs ataxia: Autosomal recessive;816 years; Ataxia,
nystagmus, dysarthria, spasticity, areflexia, proprioceptive
impairment, diabetes mellitus, optic atrophy, cardiac abnormalities.
Usually chairbound by age 20
b. Ataxia telangiectasia: Autosomal recessive; Childhood;
Progressive ataxia, athetosis, telangiectasia on conjunctivae,
impaired DNA repair, immune deficiency, tendency to malignancies
c. Abetalipoproteinaemia: Autosomal recessive Childhood
Steatorrhoea, sensorimotor neuropathy, retinitis pigmentosa,
malabsorption of vitamins A, D, E, K, cardiomyopathy
d. Spinocerebellar ataxia (types 130): Autosomal dominant;
Childhood to middle age Progressive ataxia, some types have
associated retinitis pigmentosa, pyramidal tract abnormalities,
peripheral neuropathy and cognitive deficit
Investigations:
a. These should be guided according to the differential diagnosis based upon
the initial assessment. This may include:
a. Blood tests - full blood count, liver function tests, cholesterol, protein
electrophoresis, copper and caeruloplasmin, immunoglobulins and
glycoproteins
b. EEG
c. EMG
d. Imaging - MRI is the modality of choice
Management and prognosis
This depends upon the underlying cause.

BRAINSTEM REFLEXES:
The brain stem consists of the medulla, pons, and mesencephalon. It contains
nuclei of CN III to CN XII. In one sense, it is an extension of the spinal cord
upward into the cranial cavity because it contains motor and sensory nuclei
that perform motor and sensory functions for the face and head regions in the
same way that the spinal cord performs these functions from the neck down.
But in another sense, the brain stem is its own master because it provides
many special control functions, such as the following:
1. Control of respiration
2. Control of the cardiovascular system
3. Partial control of gastrointestinal function
4. Control of many stereotyped movements of the body

39
5. Control of equilibrium
6. Control of eye movements
Brainstem reflexes:
1. Direct pupillary reaction to light. The iris constricts when bright light is
shone upon the retina (afferent CN II; efferent ipsilateral CN III).
2. Consensual pupillary reaction to light. Light stimulation of one retina
produces constriction of the contralateral pupil (afferent CN II; efferent
contralateral CN III).
3. Ciliospinal reflex. Pinching the skin on the back of the neck causes
papillary dilatation (afferent cervical somatic nerves; efferent-cervical
sympathetic chain).
4. Corneal reflex. Touching the cornea causes blinking of the eyelids
(afferent CN V; efferent CN VII).
5. Jaw reflex. When the mouth is partially opened and the muscles relaxed,
tapping the chin causes the jawto close. The reflex center is in the midpons (afferent CN V; efferent CN V).
6. Gag reflex. Gagging occurs when the pharynx is stroked. The reflex
center is in the medulla (afferent CN IX, -X; efferent CN IX, -X).
7. Oculocephalic reflex : The presence of extraocular muscle palsies assists
in localizing an intracranial lesion. After confirming the absence of neck
injury, open the eyelids and quickly turn the head from side to side. The
eyes of the comatose patient with cerebral damage turn in the opposite
direction in a conjugatemovement if the brainstem is intact (dolls eyes).
This oculocephalic reflex is lost with lesions of the pons or midbrain.
8. Oculovestibular reflex : Caloric studies, in which you test an
oculovestibular reflex, provide information of similar significance. These
are performed by irrigating the ear canal with 30 to 50 mL of ice water and
noting that, with cerebral dysfunction and an intact brainstem, a tonic
conjugate deviation of the eyes toward the cold ear lasts for 30 to 120
seconds.

CAUSES OF PARTIAL SEIZURE:


Seizures are categorized based on presentation and etiology. A generalized
seizure involves all areas of the brain (both hemispheres), whereas a partial
(focal) seizure involves only one area of the brain.
Partial seizures can be further classified as simple (i.e., no loss of consciousness)
or complex (i.e., loss of consciousness).
Classification of partial seizures:
Type
Simple partial (Consciousness
present)

B. Complex partial (Consciousness


impaired)

Etiology:

Subtypes
1. Motor
2. Sensory
3. Autonomic
4. psychic
1. Simple partial onset, followed by
impairment of consciousness
2. With impairment of consciousness at
onset

40
a. Idiopathic
1.
Benign Rolandic epilepsy of childhood
2.
Benign occipital epilepsy of childhood
b. Focal structural lesions
1.
Genetic
1. Tuberous sclerosis
2. von HippelLindau disease
3. Neurofibromatosis
4. Cerebral migration abnormalities
c. Infantile hemiplegia
d. Dysembryonic
1.
Cortical dysgenesis
2.
SturgeWeber syndrome
e. Mesial temporal sclerosis (associated with febrile convulsions)
f. Cerebrovascular disease
1.
Intracerebral haemorrhage
2.
Cerebral infarction
3.
Arteriovenous malformation
4.
Cavernous haemangioma
g. Tumours (primary and secondary)
h. Trauma (including neurosurgery)
i. Infective
1.
Cerebral abscess (pyogenic)
2.
Toxoplasmosis
3.
Cysticercosis
4.
Tuberculoma
5.
Subdural empyema
6.
Encephalitis
7.
Human immunodeficiency virus (HIV)
j. Inflammatory
1.
Sarcoidosis
2.
Vasculitis
Treatment:
First line: Carbamazepine
Second line:
Lamotrigine
Sodium valproate
Levetiracetam
Topiramate

TRIGEMINAL NEURALGIA:
1. This is characterised by lancinating pain in the second and third divisions
of the trigeminal nerve territory, usually in patients over the age of 50
years.
2. Pathophysiology
a. Trigeminal neuralgia is thought to be caused by an aberrant loop of
the cerebellar arteries compressing the trigeminal nerve as it enters
the brain stem. Other compressive lesions, usually benign, are
occasionally found.
b. When trigeminal neuralgia occurs in multiple sclerosis, there is a
plaque of demyelination in the trigeminal root entry zone.
3. Clinical features

41
a. The pain is severe and very brief but repetitive, causing the patient
to flinch as if with a motor tic (hence the French term for the
condition, tic douloureux).
b. It may be precipitated by touching trigger zones within the
trigeminal territory, by cold wind blowing on the face, or by eating.
c. Physical signs are usually absent.
d. Similar symptoms may occur in multiple sclerosis or, rarely, with
other brain-stem lesions, in which case there may be sensory
changes in the trigeminal nerve territory (and possibly other brainstem symptoms and signs).
e. There is a tendency for the condition to remit and relapse over many
years.
4. Management
a. The pain usually responds to carbamazepine in doses of up to 1200
mg daily.
b. In patients who cannot tolerate carbamazepine, gabapentin or
pregabalin may be effective.
c. Various surgical treatments are available, the simplest of which is
injection of alcohol or phenol into a peripheral branch of the nerve.
d. Probably more effective is making a radiofrequency lesion in the
nerve near the Gasserian ganglion.
e. Alternatively, the vascular compression of the trigeminal nerve can
be relieved through a posterior craniotomy, often with substantial
success.

WASTING OF SMALL MUSCLES OF HAND:


1. In the hand, the median nerve supplies the lateral two lumbricals,
opponens pollicis, abductor pollicis brevis, and flexor pollicis brevis; the
remainder are served by the ulnar nerve.
2. Wasting of the interossei (prominent guttering of the back of the hand), of
the web space between thumb and index finger, and softening and
flattening of the hypothenar eminence with sparing of abductor pollicis
brevis indicates an ulnar nerve lesion.
3. Isolated wasting of abductor pollicis brevis indicate median nerve lesion in
carpal tunnel syndrome.
4. Global wasting of hand indicate median and ulnar nerve lesion; probably,
with damage to T1 root.
5. More extensive arm wasting may indicate any of the following:
a. syringomyelia
b. bilateral, symmetrical wasting indicate peripheral neuropathy.
c. Spinal cord lesions at T1 level: cervical spondylosis, neurofibroma
d. Motor neurone disease

42
e. Brachial plexus lesions:
1. Trauma,
2. Irradiation,
3. Infiltration
4. Thoracic outlet obstruction
5. Klumpke's paralysis
Myopathy:
1. Distal myopathy
2. Myotonic dystrophy (forearms are more often affected than the
hands)
Trophic disorders:
1. Disuse eg arthritis
2. Ischaemia eg vasculitis
3. Shoulder-hand syndrome
TREATMENT OF BACTERIAL MENINGITIS:2008 MENTION CAUSES OF
MENINGITIS.2010 AETIOLOGY OF PYOGENIC MENINGITIS 2009
1. Causes of bacterial meningitis
Age of onset
Neonate

Common
Escherichia coli,
monocytogenes
Proteus
Group B streptococci

Less common
Listeria

Pre-school child
Haemophilus influenzae Mycobacterium
tuberculosis
Neisseria meningitidis
Streptococcus pneumoniae
Older child and adult
Neisseria meningitidis
Listeria
monocytogenes
Streptococcus pneumoniaMycobacterium
tuberculosis
Staphylococcus aureus
(skull fracture)
Haemophilus
influenzae
2. Other causes of meningitis:
1. Bact: Brucella
2. Viruses
1.
Enteroviruses (echo,Coxsackie, polio)
2.
Mumps
3.
Influenza
4.
Herpes simplex
5.
Varicella zoster
6.
EpsteinBarr

43
HIV
Lymphocytic choriomeningitis
Mollarets meningitis (herpes simplex virus type 2)
3. Protozoa and parasites
1.
Toxoplasma
2.
Amoeba
3.
Cysticercus
4. Fungi
1.
Cryptococcus neoformans
2.
Candida
3.
Histoplasma
4.
Blastomyces
5.
Coccidioides
6.
Sporothrix
5. Non-infective (sterile)
1.
Malignant disease
1.
Breast cancer
2.
Bronchial cancer
3.
Leukaemia
4.
Lymphoma
2.
Inflammatory disease (may be recurrent)
1.
Sarcoidosis
2.
SLE
3.
Behets disease
Treatment of meningitis:
1. Initial Management Steps:
a. Bacterial meningitis is a neurologic emergency; progression to
more severe disease reduces the patients likelihood of a full
recovery
b. A blood culture and lumbar puncture should be performed
immediately after ruling out increased intracranial pressure by
CT scan brain
c. Begin antimicrobial therapy as soon as possible.
2. Empiric therapy:
a. Patients with a typical meningococcal rash
Benzylpenicillin 2.4 g i.v. 6-hourly
b. A dults aged 1850 years without a typical meningococcal
rash
Cefotaxime 2 g i.v. 6-hourly or
Ceftriaxone 2 g i.v. 12-hourly
c. Patients in whom penicillin-resistant pneumococcal
infection is suspected
As for (2) but add:
Vancomycin 1 g i.v. 12-hourly or
Rifampicin 600 mg i.v. 12-hourly
d. A dults aged over 50 years and those in whom Listeria
monocytogenes infection is suspected (brain-stem signs,
immunosuppression, diabetic, alcoholic)
As for (2) but add:
Ampicillin 2 g i.v. 4-hourly or
Co-trimoxazole 50 mg/kg i.v. daily in two divided
doses
e. Patients with a clear history of anaphylaxis to -lactams
Chloramphenicol 25 mg/kg i.v. 6-hourly plus
7.
8.
9.

44
Vancomycin 1 g i.v. 12-hourly
3. Chemotherapy of bacterial meningitis when the cause is known

Pathogen
N. meningitides
Strep.
Pneumonia
(sensitive to blactams,
Strep.
pneumoniae
resistant to lactams
H. influenzae
Listeria
monocytogenes
Strep. suis

Regimen of choice
Benzylpenicillin2.4 g i.v. 4-hourly for 57 days
Cefotaxime 2 g i.v.
6-hourly or ceftriaxone 2 g i.v.12-hourly for 10
14 days
As for sensitive strains but add vancomycin
1 g i.v. 12-hourly or rifampicin 600 mg i.v.12hourly
Cefotaxime 2 g i.v.6-hourly or ceftriaxone 2 g i.v.
12-hourly for 1014 days
Ampicillin 2 g i.v. 4-hourly plus gentamicin 5
mg/kg i.v. daily
Cefotaxime 2 g i.v. 6-hourly or ceftriaxone 2 g
i.v.
12-hourly for 1014 days

Corticosteroids:
a. Steroids are thought to improve the outcome in patients with
bacterial meningitis by suppressing the inflammatory response
that occurs with bacterial cell lysis.
b. Therefore, steroids must be given prior to or concurrent with the
first dose of parenteral antibiotics. Supplementary antiinflammatory therapy with high-dose dexamethasone has been
the subject of several recent high-quality trials.

MENTION SIGNS OF MENINGIAL IRRITATION. 2010

1. Meningism or Meningeal irritation consists of headache, photophobia


and stiffness of the neck, often accompanied by other signs
2. Meningism is not specific to meningitis and can occur in patients with
subarachnoid haemorrhage.
3. Neck stiffness is the most important sign of meningitis. When the
neck is flexed, the inflamed nerve roots and meninges of the cervical
region get stretched. This causes protective muscle spasm manifesting
as neck stiffness.
4. Kernigs Sign: the maneuver is usually performed with the patient
supine with hips and knees in flexion. Extension of the knees is
attempted: the inability to extend the patients knees beyond 135
degrees without causing pain constitutes a positive test for Kernigs
sign.
5. Brudzinskis Sign: With the patient supine, the physician places one
hand behind the patients head and places the other hand on the
patients chest. The physician then raises the patients head (with the
hand behind the head) while the hand on the chest restrains the patient
and prevents the patient from rising. Flexion of the patients lower

45
extremities (hips and knees) constitutes a positive sign. Brudzinskis
neck sign has more sensitivity than Kernigs sign.
6. Brudzinskis contralateral reflex sign. Brudzinskis contralateral
reflex sign has two components: the identical and reciprocal
contralateral reflux. The patients hip and knee are passively flexed on
one side; if the contralateral leg bends in reflex, identical contralateral
reflex is demonstrated. Reciprocal contralateral reflex occurs when the
leg that has flexed in response to the passive flexion of the
contralateral hip and knee begins to extend passively.
7. Tripod sign, also known as the "Amoss's sign," is a useful sign of
meningeal irritation. The patient with meningeal irritation tries to sit up
by supporting himself with his hands placed far behind him in the bed
(like a tripod)

LUMBAR PUNCTURE: (S.N)


1. Examination of the cerebrospinal fluid (CSF) is essential in confirming the
diagnosis of meningitis, encephalitis, and subarachnoid hemorrhage and is
often helpful in evaluating demyelinating, degenerative, and collagen
vascular diseases and the presence of tumor cells within the subarachnoid
space.
2. It is the test of choice to diagnosis pseudotumor cerebri.
3. Procedure:
a. The skin is thoroughly prepared with a cleansing agent, and the
patient is placed in the lateral recumbent position.
b. The physician should wear a mask and be gowned and gloved; the
patient should be draped.
c. The neck and legs of the patient are flexed by an assistant to
enlarge the intervertebral spaces.
d. The ideal interspace for lumbar puncture (LP) is L3-L4 or L4-L5,
which is determined by drawing an imaginary horizontal line from
one anterior superior spine of the ilium to the other.
e. The skin and underlying tissue are anesthetized with a local
anesthetic
f. A 22-gauge, 12 in, sharp, beveled spinal needle with stylet is
introduced into the midsagittal plane, directed slightly in the
cephalic direction. The stylet is removed frequently as the needle is
slowly advanced to determine whether CSF is present.
g. A pop is felt as the needle penetrates the dura and enters the
subarachnoid space. A manometer and a three-way stopcock may
be attached to obtain an opening pressure.
4. Contraindications for performing an LP include: (1) elevated ICP owing to
a suspected mass lesion of the brain or spinal cord, (2) symptoms and
signs of pending cerebral herniation in a child with probable meningitis, (3)
critical illness (on rare occasions), (4) skin infection at the site of the LP,
and (5) thrombocytopenia.
5. Complications:
1.1. Transtentorial herniation or herniation of the cerebellar tonsils(In ICT)
2. May precipitate cardiorespiratory arrest
3. headache, which is thought to be due to reduced CSF pressure.
4. Transient radicular pain

46
5. Infections such as meningitis
6. Haemoorhage in in the subarachnoid or subdural space in cases of
thrombocytopenia or disseminated intravascular coagulation
6. Normal CSF:
a. The opening pressure in the recumbent and relaxed position
averages 100 mm of fluid; the range in the flexed lateral decubitus
position is 60180 mm of fluid.
b. The most common cause of an elevated opening pressure is a
crying, uncooperative, and struggling patient.
c. It is the color of water. Cloudy CSF results from an elevated white
blood cell (WBC) or red blood cell (RBC) count.
d. Normal CSF contains up to 5/mm3WBCs, and a newborn may have as
many as 15/mm3. Polymorphonuclear (PMN) cells are always
abnormal in a child, but 12/mm3may be present in a normal
neonate.
7. Abnormal findings:
a. An elevated polymorph count suggests bacterial meningitis
b. CSF lymphocytosis indicates aseptic, tuberculous, or fungal
meningitis;
8. CSF protein:
a. The normal CSF protein ranges from 10 to 40 mg/dL
b. The CSF protein may be elevated in many processes, including
infectious, immunologic, vascular, and degenerative diseases as
well as tumors of the brain and spinal cord.
c. The CSF protein is increased after a bloody tap by 1 mg/dL for
every 1,000 RBCs/mm3.
d. Elevation of CSF immunoglobulin G (IgG), which normally represents
10% of the total protein, is observed in subacute sclerosing
panencephalitis, postinfectious encephalomyelitis, and in some
cases of multiple sclerosis.
e. If the diagnosis of multiple sclerosis is suspected, the CSF should be
tested for the presence of oligoclonal bands.
9. CSF Glucose:
a. The CSF glucose content is about 60% of the blood glucose.
b. Hypoglycorrhachia is found in association with diffuse meningeal
disease, particularly bacterial and tuberculous meningitis.
10.Bacteriology:
a. A Gram stain of the CSF is essential in the investigation of suspected
bacterial meningitis;
b. an acid-fast stain or India ink preparation is used if tuberculous or
fungal meningitis is a possibility.
c. The fluid is placed on appropriate culture media based on the
clinical findings and on the CSF analysis.
d. Further, as indicated by the history and clinical examination, a
portion of the CSF should be sent for viral (PCR or antibody) studies
(HSV1 and 2, West Nile, and enteroviruses).
11.Antigens in CSF:
a. The CSF may also be examined for specific antigens (latex
agglutination for suspected meningitis) and metabolites in
investigation of a series of metabolic diseases (lactate, amino acids,
enolase determination).

47
12.Traumatic LP:
a. Normal CSF contains no RBCs. The presence of RBCs indicates a
traumatic tap or a subarachnoid hemorrhage. Bloody CSF should be
centrifuged immediately. The supernatant of a bloody tap is clear,
but it is xanthochromic in the presence of a subarachnoid
hemorrhage. Progressive clearing of bloody CSF is noted during
collection of the fluid in the case of a traumatic tap.

WERNICKES ENCEPHALOPATHY: AGAIN AUG 2008 PII


1. Thiamine (vitamin B-1) deficiency can result in Wernicke

encephalopathy, a serious neurologic disorder.


2. Thiamine plays a vital role in the metabolism of carbohydrates.

Thiamine is a cofactor for several essential enzymes in the Krebs cycle


and the pentose phosphate pathway, including alpha-ketoglutarate
dehydrogenase, pyruvate dehydrogenase, and transketolase. In the
setting of thiamine deficiency, thiamine-dependent cellular systems
begin to fail, resulting eventually in cell death. Because thiaminedependent enzymes play an essential role in cerebral energy utilization.
Thiamine deficiency may propagate brain tissue injury by inhibiting
metabolism in brain regions with higher metabolic demands and high
thiamine turnover
3. Dr Carl Wernicke, a Polish neurologist, described it in 1881 as a triad of
acute mental confusion, ataxia, and ophthalmoplegia.
4. Thiamine deficiency is characteristically associated with chronic
alcoholism, because it affects thiamine uptake and utilization.
5. However, Wernicke encephalopathy may develop in nonalcoholic
conditions, such as prolonged starvation, hyperemesis gravidarum,
bariatric surgery, and HIV and can even develop in healthy infants
given the wrong formulas
6. Korsakoff amnestic syndrome is a late neuropsychiatric manifestation of
Wernicke encephalopathy with memory loss and confabulation;
sometimes, the condition is referred to as Wernicke-Korsakoff syndrome
or psychosis.
7. This organic brain disorder results from damage to the mamillary
bodies, dorsomedial nuclei of the thalamus and adjacent areas of
periventricular grey matter. It is caused by a deficiency of thiamin
(vitamin B1 ), which is most commonly caused by long-standing heavy
drinking and an inadequate diet.
8. Without prompt treatment, the acute presentation of Wernickes
encephalopathy (nystagmus,ophthalmoplegia, ataxia and confusion)
can progress to the irreversible deficits of Korsakoffs syndrome (severe
short-term memory deficits and confabulation).
9. In those who die in the acute stage, microscopic examination of the
brain shows hyperaemia, petechial haemorrhages and astrocytic
proliferation.
10. WernickeKorsakoff disease
1. This presents with an acute confusional state (Wernickes
encephalopathy) and brain-stem abnormalities such as ataxia,
nystagmus and extraocular muscle weakness, particularly
affecting the lateral rectus muscle.

48
2. The condition is caused by deficiency of thiamin (vitamin B1) and

is often associated with alcoholism. However, other causes


include generalised malnutrition, malabsorption or protracted
vomiting (as in hyperemesis gravidarum).
11. Treatment consists of intravenous thiamin (in the form of Pabrinex,2
vials 8-hourly for 48 hours) initially, followed by oral (100 mg 8-hourly),
in addition to treating the underlying cause.

MENTION TWO CHANGES IN CNS WITH AGEING AND THEIR


CORRESPONDING CONSEQUENCES.
1.
1.
2.
3.
4.
5.
6.

CNS Changes with ageing Clinical consequences


Neuronal loss:
Increased risk of delirium
Cochlear degeneration:
Presbyacusis/high-tone hearing loss
Increased lens rigidity: Presbyopia/abnormal near vision
Lens opacification:
Cataract
Anterior horn cell loss: Muscle weakness and wasting
Dorsal column loss:
Reduced position and vibration sense
Slowed reaction times:Increased risk of falls

DRAW AND LABEL A DIAGRAM OF CIRCLE OF WILLIS:

HORMERS SYNDROME.

49
1. Damage to the ipsilateral sympathetic nervous system
2.

Congenital or iatrogenic.

3.

Pancoast tumor or thyrocervical venous dilatation.

4.

Signs found in all patients on affected side of face include;


1. Ptosis
2. Miosis
3. Anhidrosis
4. Enophthalmos
5. Sometimes there is flushing of the face

5. Causes:
1. Transection of the cervical spinal cord (hypothalamospinal
pathway)
2. Compression of the sympathetic chain by a lung tumor
(Preganglionic lesions)
3. A tumor in the cavernous sinus (Postganglionic lesions)
6. A lesion anywhere along the long sympathetic pathway results in a

typical Horner's syndrome with miosis and ptosis.


7. Pathway:
1. Hypothalamus medullary ciliospinal center spinal

ganglion superior cervical ganglion internal carotid artery


plexus in cavernous sinus abducence nerve optholmic
division of C5 nasociliary branch in orbit
8. The ptosis in Horner's syndrome is usually slight, due to paralysis of
the sympathetically innervated smooth muscle (Mller's muscle) in
the upper eyelid.
9. Similar smooth muscle fibres in the lower eyelid are denervated,
leading to a slight elevation of the lower lid, producing an upsidedown ptosis. Combined, these result in a narrowed palpebral fissure
and an apparent enophthalmos.
10. The miosis is due to complete or partial sympathetic denervation of
the iris dilator muscle, leading to constriction of the iris sphincter,
producing a small pupil.
11. The weakness of the dilator muscle is greatest in the dark when the
anisocoria is most apparent, and may be almost absent in the light.
12. Depigmentation of the affected iris is rarely observed in acquired
Horner's disease, although hypochromia of the iris is a common
finding in the congenital form.

50
13. Horner's syndrome is also associated with characteristic vasomotor

and sudomotor changes on the affected side of the face, such as


loss of sweating (anhidrosis) and occasionally facial flushing. These
changes are most frequently observed following preganglionic
lesions, since the fibres for sweating pass onto the external carotid
artery from the superior cervical ganglion.
14. Aetiology of Horner's syndrome
1.
2.
3.
4.
5.
6.
7.

Lateral medullary infarction


Intracranial tumour
Cerebral haemorrhage
Trauma (including surgery)
Multiple sclerosis
Thoracic and neck tumour
Intracranial tumour (cavernous sinus)

GAIT. 2010 ; FOUR GAIT DISORDERS 2011


1. Gait: Gait is the attitude of a person in the upright position.
2. Many neurological disorders are associated with an abnormal gait and
observing a patient walk can be very informative and assist in coming
to a neurological diagnosis. It is also an important element of assessing
disability.
3. Various patterns of weakness, loss of coordination and proprioceptive
sensory loss produce an abnormal gait.
4. Neurogenic gait disorders need to be distinguished from those due to
skeletal abnormalities, usually characterised by pain producing an
antalgic gait, or limp.
5. Gaits that do not fit either pattern may be due to psychiatric disorders
and are usually incompatible with any anatomical or physiological
deficit.
6. Pyramidal gait:
a. Upper motor neuron lesions cause a so-called pyramidal gait
in which the upper limb is held in flexion while the ankle joint
in the lower limb is kept relatively extended. This causes a
tendency for the toes to strike the ground while walking and in
an attempt to overcome this, the leg is swung outwards at the
hip (circumduction).
b. in paraparesis both lower limbs swing slowly from the hips in
extension and are dragged stiffly over the ground. This can
often be heard as well as seen.
7. Foot drop
a. If there is a lower motor neuron lesion affecting the lower
limb, weakness of ankle dorsiflexion occurs, which makes a
slapping noise as it hits the ground.
b. If the distal weakness is severe, the foot will have to be lifted
higher at the knee to allow room for the inadequately
dorsiflexed foot to swing through, resulting in a high-stepping
gait.
8. Myopathic gait
a. During walking, alternating transfer of the bodys weight
through each leg requires careful control of hip abduction by

51
the gluteal muscles. In proximal muscle weakness, usually
caused by muscle disease, the hips are not properly fixed by
these muscles and trunk movements are exaggerated,
producing a rolling or waddling gait.
9. Ataxic gait
a. An ataxic gait can occur as the result of lesions in the
cerebellum, vestibular apparature or peripheral nerves.
b. Patients with lesions of the central portion of the cerebellum
(the vermis) walk with a characteristic broad-based gait like a
drunken sailor (cerebellar function is particularly sensitive to
alcohol). Patients with acute vestibular disturbances walk in a
similar fashion, but the accompanying vertigo distinguishes
them from those with cerebellar lesions. Less severe degrees
of cerebellar ataxia can be detected by asking the patient to
walk heel to toe; patients with vermis lesions are unable to do
this. Defects in proprioception can also cause an ataxic gait.
c. The impairment of joint position sense makes walking
unreliable, especially in poor light. The feet tend to be placed
on the ground with greater emphasis, presumably in an
attempt to increase what proprioceptive input is available.
This results in a stamping gait which is often combined with
foot drop when caused by a peripheral neuropathy, but it can
also occur in disorders of the dorsal columns in the spinal
cord.
10.
Apraxic gait
a. In an apraxic gait, there is normal power in the legs, no
cerebellar ataxia and no proprioception loss, and yet the
patient still cannot formulate the motor act of walking. In this
higher cerebral dysfunction, the feet appear stuck to the floor
and the patient cannot walk, even though leg movement is
normal on the examination couch. Gait apraxia occurs in
bilateral hemisphere disease such as normal pressure
hydrocephalus and diffuse frontal lobe disease.
11.
Marche petits pas
a. This gait is characterised by small, slow steps and marked
instability. This looks different from the Festinant gait of
Parkinsons disease in that it does not have the variable pace
and freezing. The usual cause is multiple small-vessel
cerebrovascular disease and there are often signs of bilateral
upper motor neuron disease.
12.
Extrapyramidal gait
a. Patients with Parkinsons disease and other extrapyramidal
diseases have difficulty initiating walking and difficulty
controlling the pace of their gait. Patients may get stuck whilst
trying to start walking or when walking through doorways
(freezing). Once started, they may shuffle and have
problems controlling the speed of their walking and
sometimes have difficulty stopping. This produces the
festinant gait: initial stuttering steps that quickly increase in
frequency while decreasing in length.

52
13.

Spastic or scissor gait


a. The legs in adduction at the hip and the thighs rub against
each other
b. This is typically seen in cerebral diplegia, a form of cerebral
palsy.

MANAGEMENT OF MIGRAINE; MIGRAINE- FEB 2010 PAPER II


1. Incidence: 20% of females and 6% of males
2. Usually presents before the age of 40
3. Pathophysiology
a. The cause of migraine is incompletely understood
b. the disorder may be due to abnormal function of ion channels, which in
some cases is genetically determined.
c. The headache is associated with vasodilatation of extracranial vessels
d. Family history is common in migraine, suggesting a genetic predisposition,
e. The great female preponderance and the tendency for some women to
have migraine attacks at certain points in their menstrual cycle hint at
hormonal influences.
f. Contraceptive pill appears to exacerbate migraine in many patients, and to
increase the small risk of stroke in patients who suffer from migraine with
aura.
g. In some patients there are identifiable dietary precipitants such as cheese,
chocolate or red wine.
h. Migraine attack often occurs after the period of stress, so that some
patients tend to have attacks at weekends or at the beginning of a holiday.
i. Clinical features
i. Symptom: triad of paroxysmal headache, nausea and/or vomiting,
and an aura of focal neurological events (usually visual). Patients
with all three of these features are said to have migraine with aura
(classical migraine).
ii. Those with paroxysmal headache (with or without vomiting) but no
aura are said to have migraine without aura (common migraine).
iii. A classical migraine attack starts irritability followed by the aura,
and then a severe, throbbing, hemicranial headache with
photophobia and vomiting.
iv. During the headache phase, patients prefer to be in a quiet,
darkened room and to sleep. The headache may persist for several
days.
v. Aura: The aura most often takes the form of visual fortification
spectra or a sensory aura
vi. If the dominant hemisphere is involved, the patient may also
experience transient aphasia.
vii. Limb weakness can occur in migraine and is termed hemiplegic
migraine.
viii. In a smaller number of patients, the symptoms of the aura do not
resolve, leaving more permanent neurological disturbance
(complicated migraine).
j. Management
i. Identification and avoidance of precipitants or exacerbating factors
(such as the contraceptive pill) may prevent attacks.

53

4.

5.

6.

7.

8.

ii. Treatment of an acute attack consists of simple analgesia with


aspirin or paracetamol, often combined with an antiemetic such as
metoclopramide or domperidone.
iii. Long-term use of codeine-containing analgesic preparations should
be avoided.
iv. Severe attack is treated with one of the triptans (e.g. sumatriptan),
5-HT agonists that are potent vasoconstrictors of the extracranial
arteries. These can be administered orally, sublingually, by
subcutaneous injection or by nasal spray.
v. Ergotamine preparations should be avoided since they easily lead to
dependence. This is less likely to happen with the triptans, but it can
occur.
vi. If attacks are frequent, they can often be prevented with;
1. Propranolol (80160 mg daily, in a sustained release
preparation),
2. Tricyclic such as amitriptyline (1050 mg at night),
3. Sodium valproate (300600 mg/day)
4. Topiramate (50100 g/day).
vii. Women should be warned that the small risk of ischaemic stroke
attributable to taking oral contraception is increased if they have
migraine with aura, especially if they also smoke.
Cluster headache: Cluster headaches (also known as migrainous neuralgia)
are 1050 times less common than migraine. There is a 5:1 predominance of
males and onset is usually in the third decade.
Pathophysiology
a. The cause is unclear. Functional imaging studies have suggested abnormal
neuronal activity in the hypothalamus.
b. Patients are usually heavy smokers with a higher than average alcohol
consumption.
Clinical features
a. The characteristic presentation is with periodic, severe, unilateral
periorbital pain accompanied by unilateral lacrimation, nasal congestion
and conjunctival injection, often with the other features of Horners
syndrome.
b. The pain, whilst being very severe, is characteristically brief (3090
minutes). Typically, the patient develops these symptoms at a particular
time of day (often in the early hours of the morning).
c. The syndrome may occur repeatedly for a number of weeks, followed by a
respite for a number of months before another cluster occurs.
Management
a. Acute attacks can usually be halted by subcutaneous injections of
sumatriptan or by inhalation of 100% oxygen, but other migraine therapies
are ineffective, probably because of the brevity of the individual attacks.
b. Preventative therapy with the agents used for migraine is often ineffective
but attacks can be prevented in some patients by verapamil (80120 mg 8hourly), methysergide (410 mg daily, for a maximum of 3 months only) or
short courses of oral corticosteroids.
Patients with severe and debilitating clusters can be helped with lithium
therapy, although the usual precautions concerning the use of this drug
should be observed.

54

PERIPHERAL NEUROPATHY (S.N) FEB 2013


Introduction:
1. The peripheral nerves consist of bundles of long neuronal axons as they
exit the central nervous system (CNS). Some peripheral nerves are
wrapped in a myelin sheath generated by Schwann cells, whereas others
are unmyelinated. Peripheral nerves serve different motor, sensory, and
autonomic functions.
2. The term peripheral neuropathy is used to describe symmetric and
universal damage to adjacent nerves. The damage and clinical
manifestations are usually located distally with a proximal progression.
3. Numerous inherited and acquired pathological processes may affect
peripheral nerves, targeting either the nerve roots (radiculopathy), the
nerve plexuses (plexopathy) and/or the individual nerves themselves
(neuropathy)
4. Cranial nerves 312 share the same tissue characteristics as peripheral
nerves elsewhere and are subject to the same range of diseases.
5. Nerve fibres of different types (motor, sensory or autonomic) and of
different sizes may be variably involved.
6. Disorders may be primarily directed at the axon, the myelin sheath
(Schwann cells) or the vasa nervorum
7. Dosorders may be:

a.
Acute
b.
Chronic
c.Focal (affecting a single nerve: mononeuropathy),
d.
Multifocal (several nerves: mononeuropathy multiplex)
e.
Generalised (polyneuropathy).
Etiology:
1. Systemic disorders and toxins
a. Metabolic/endocrine:
1.
Diabetes mellitus
2.
Chronic renal failure
3.
Hypothyroidism
4.
Porphyria

5.

Acromegaly
b. Toxic:
i. Alcoholism
ii. Chronic liver disease
iii. Drugs
iv. Lead
v. Radiation
vi. Arsenic
vii. Mercury
c. Immune-mediated/inflammatory:
i. Systemic vasculitis
ii. SLE
iii. Rheumatoid arthritis
iv. Sjgrens disease
v. Cryoglobulinaemia

55

vi. Paraproteinaemia
vii. Coeliac disease
viii. Sarcoidosis
ix. Primary amyloidosis
d. Infective:
i. HIV
ii. Lyme disease
e. Neoplastic
i. Lymphoma
ii. Carcinoma (infiltration and paraneoplastic)
iii. Myeloma
f. Vitamin deficiencies
i. Vitamin B12
ii. Thiamin
iii. Pyridoxine
iv. Folic acid
v. Vitamin E
i. Pantothenic acid
2. Drugs causing peripheral neuropathy:
a. Cardiac:
i. Amiodarone
ii. Statins
iii. Hydralazine
b. Chemotheraputics:
i. Cisplatin
ii. Thalidomide
iii. Paclitaxel
iv. Vincristine
c. Anti infective:
i. Chloramphenicol
ii. Isoniazid
iii. Nitrofurantoin
iv. Ethambutol
v. Metronidazole
vi. Suramin
d. Others
i. Gold
ii. Disulfiram
iii. Pyridoxine
iv. Tacrolimus
v. Colchicine
vi. Phenytoin
3. Signs and symptoms:
a. neuropathies often present with burning pain, lightning-like or lancinating pain
or uncomfortable paresthesias (dysesthesias). Patients may complain of pain w
innocuous stimuli such as sheets rubbing over their feet (allodynia).
b. They may also describe a tight, band-like sensation around the ankles or wrist
c. Sensory symptoms include tingling or paresthesias, increased sensation in affe
areas (hypesthesia), and numbness or reduced sensation.
d. Dying-back (distal symmetric axonal) neuropathies initially involve the tips of t
and progress proximally in a stocking-glove distribution.
e. Multifocal neuropathies, such as mononeuritis multiplex caused by polyarteriti
may result in sensory abnormalities in specific nerve or root distributions.
f. Motor symptoms such as weakness and wasting also commence distally in a d

56

neuropathy.
g. Common complaints are tripping on the toes and loss of grip strength.
h. The patient may have cramps or fasciculations.
i. Peripheral neuropathy can present as restless leg syndrome.
j. Proximal involvement may result in difficulty climbing stairs, getting out of a c
and swallowing, and in dysarthria.

4. Investigation of peripheral neuropathy


a. Haematology
i. Full blood count
ii. ESR
iii. B12 and folate
b. Biochemistry
i. Urea, electrolytes, calcium
ii. Creatinine
iii. Liver function tests
iv. Blood glucose tolerance test/HbAlc
v. Thyroid function tests
vi. Plasma protein electrophoresis
c. Immunology
i. VDRL
ii. Serum autoantibodies (antinuclear factor, dsDNA,rheumatoid
factor, extractable nuclear antigens
d. Genetic screening tests (hereditary neuropathies, Friedreichs
ataxia)
e. Vitamin assays (e.g. vitamin E) Phytanic acid (Refsums disease)
f. Nerve conduction/EMG
g. Nerve biopsy (sural nerve)
5. Treatment:
a. Removal of offending agent: toxins or medications
a. correcting a nutritional deficiency: B1;B12
b. corticosteroid therapy for immune-mediated neuropathy: GBS
c. Acute inflammatory neuropathies require more urgent and
aggressive management with intravenous immunoglobulin or
plasmaphereis: GBS
d. neuropathic pain: gabapentin, topiramate, carbamazepine,
pregabalin
9

EEG: FEB 2013


a. The electroencephalogram (EEG) is used to detect electrical activity arising in
the cerebral cortex.
b. The EEG involves placing an array of electrodes on the scalp to provide spatial
information, and recording the waveforms which are distinguished by their
amplitude and frequency.
c. When the eyes are shut, the most obvious frequency over the occipital cortex
is 813 Hz; this is known as alpha rhythm, and disappears when the eyes are
opened.
d. Other frequency bands seen over different parts of the brain in different
circumstances are beta (faster than 13/s), theta (48/s) and delta (slower than
4/s).
e. Lower frequencies predominate in the very young and during sleep.
f. Various diseases cause abnormalities in the EEG. These may be continuous or
episodic, focal or diffuse.

57
g. Examples of continuous abnormalities include a global increase in fast
frequencies (beta) seen with sedating drugs such as benzodiazepines, or
marked focal slowing seen over a structural lesion such as a tumour or an
infarct.
h. It is useful in the investigation of patients who have disturbance of
consciousness or disorders of sleep, in the diagnosis of cerebral diseases such
as encephalitis, and in certain dementias such as sporadic CreutzfeldtJakob
disease.
i. The EEG is predominantly used in the diagnosis and assessment of epilepsy,
to distinguish the type of epilepsy and to determine the location of an
epileptic focus, particularly if surgery is contemplated. It must be stressed
that only in rare circumstances will an EEG provide unequivocal evidence of
epilepsy;
j. During an epileptic seizure, highvoltage disturbances of the background
activity (transients) can occur. These may be generalised, as in the 3 cycle/s
spike and wave of childhood absence epilepsy (petit mal), or more focal, as
in partial epilepsies.
k. it is often possible to detect epileptiform abnormalities in between seizures
in the form of spikes and sharp waves that lend support to a clinical
diagnosis.
l. The likelihood of detecting these abnormalities is enhanced by
hyperventilation, photic flicker, sleep and some drugs.
Evoked potentials
a. The EEG can also be used to study evoked potentials (EPs), which can be
measured following visual, auditory or somatosensory stimuli if the electrodes
are appropriately positioned.
b. Abnormalities of the EP indicate damage to the relevant pathway, in the form
of either a conduction delay (increased latency) or reduced amplitude, or
both.

SYMTOMS AND SIGNS OF INCREASED INTRA CRANIAL TENSION.


PAPER II AUG 2012
Definition:
1. ICP is generally measured via pressure transducers inserted directly into the
brain tissue.
2. The normal ICP is 15 mmHg:
3. Increasd if > 20 mmHg.
4. Sustained pressures > 30 mmHg are associated with a poor prognosis.
Etyiology:
1. Mass lesions (especially tumours),
2. Cerebral oedema,
3. Obstruction to CSF circulation (causing hydrocephalus)
4. Impaired CSF absorption, as in idiopathic intracranial hypertension
5. Cerebral venous obstruction.
Clinical features
1. Local effects on adjacent brain tissue (e.g. seizures, focal signs)
2. Raised intracranial pressure
a. Headache
b. Impairment of conscious level
c. Papilloedema

58
d. Vomiting, bradycardia, arterial hypertension
3. False localising signs
a. Pupillary dilatation (ipsilateral to lesion)
b. 6th cranial nerve lesion (unilateral or bilateral)
c. Hemiparesis (ipsilateral to lesion)
d. Bilateral extensor plantar responses
4. Coning:
a. Downward displacement of the temporal lobes through the
tentorium due to a large hemisphere mass may cause temporal
coning. This may stretch the 3rd and/or 6th
cranial nerves, or cause pressure on the contralateral cerebral
peduncle (causing ipsilateral upper motor neuron signs).
b. Downward movement of the cerebellar tonsils through the foramen
magnum may compress the medullatonsillar coning. This coning
may result in brain-stem haemorrhage and/or acute obstruction of
the CSF pathways. As coning progresses, the patient may adopt a
decerebrate posture and, unless rapidly treated, death almost
invariably ensues.
c. LP is contraindicated in conig.
Management of ICP:
1. The management of raised intracranial pressure is largely dictated by its
specific cause
2. ICU support may be required Prevent coughing with sedation, analgesia
and occasionally paralysis
3. Nurse with 30 head-up tilt and avoid excessive flexion of the head or
pressure around the neck that may impair cerebral venous drainage
4. Control epileptiform activity with appropriate anticonvulsant therapy; an
EEG may be necessary to ensure that this is achieved
5. Maintain good glycaemic control with blood glucose between 5.5 and 8
mmol/L (~99144 mg/dL)
6. Aim for a core body temperature of between 36 and 37C
7. Maintain sodium > 140 mmol/L using i.v. 0.9% saline
8. Avoid volume depletion or fluid overload
9. Provide ventilation aiming to reduce the PaCO2 to 44.5 kPa (~3034
mmHg) for the first 24 hours
10.Osmotic diuretic, mannitol 20% 100200 mL (0.250.5 g/kg), coupled with
volume replacement
11.Hypnotic infusion, thiopental, titrated to burst suppression on EEG
12.
Surgery: drainage of haematoma or ventricles; lobectomy,
decompressive craniectomy

CSF FEATURES OF TUBERCULOUS MENINGITIS. PAPER II FEB 2011

1.
2.
3.
4.
5.
6.

NORMAL
TBM
Pressure
50250 mm of water
Normal / increased
Colour
clear
Clear / cloudy
Red cell count
( 106/L)
04
Normal
White cell count ( 106/L)
04
505000 lymphocytes
Glucose
> 60% of blood level
Decreased
Protein
< 0.45 g/L
Increased; cob web formation

59
7. Microbiology
Nielson/auramine stain or

Sterile

8. Oligoclonal bands

Negative

Ziehl
tuberculosis culture positive
Can be positive

COMPUTERIZED TOMOGRAPHY IN STROKE. FEB 2010 P II


1. CT is the most practical and widely available method of imaging the brain.
2. It will usually exclude nonstroke lesions, including subdural haematomas
and brain tumours, and will demonstrate intracerebral haemorrhage
within minutes of stroke onset.
3. However, especially within the first few hours after symptom onset, CT
changes in cerebral infarction may be completely absent or only very
subtle.
4. Changes often develop over time, but small cerebral infarcts may never
show up on CT scans.
5. For most purposes, a CT scan performed within the first day or so is
adequate for clinical care but there are certain circumstances in which an
immediate CT scan is essential:
a. Patient on anticoagulants or with abnormal coagulation
b. Consideration of thrombolysis or immediate anticoagulation
c. Deteriorating conscious level or rapidly progressing deficits
d. Suspected cerebellar haematoma, to exclude hydrocephalus
6. Even in the absence of changes suggesting infarction, abnormal perfusion
of brain tissue can be imaged with CT after injection of contrast media
(i.e. perfusion scanning). This can be useful in guiding immediate
treatment of ischaemic stroke.
7. CT and MRI may reveal clues as to the nature of the arterial lesion. For
example, there may be a small, deep lacunar infarct indicating smallvessel disease, or a more peripheral infarct suggesting an extracranial
source of embolism.
8. In a haemorrhagic lesion, the location might indicate the presence of an
underlying vascular malformation, saccular aneurysm or amyloid
angiopathy.

PHENYTOIN
1. Phenytoin acts by decreasing the sustained repetitive firing of single
neurons by blocking sodium-dependent channels and decreasing
depolarization-dependent calcium uptake.
2. Phenytoin is used for primary and secondary generalized tonic-clonic
seizures, partial seizures, and status epilepticus.
3. The plasma half-life is 742 hr. Phenytoin has many drug interactions that
may increase or decrease other concomitantly used anticonvulsants.
Fosphenytoin, a water-soluble prodrug of phenytoin for intramuscular or
intravenous use, is available in a concentration of 75 mg/mL, equivalent to
50 mg/mL of phenytoin.

60

DRUG

SEIZURE
TYPE

Phenyto Generalized
in
tonic-clonic
(Dilantin
Partial
)
Sta.epilepicus

ORAL
DOSE

THERAPEUTI
LOADIN
C SERUM
G DOSE
SIDE EFFECTS AND TOXICITY
LEVEL
(IV)
(G/ML)

39
20
mg/kg/2 mg/kg
4 hr bid

1020

Hirsutism, gum hypertrophy,


ataxia, skin rash, StevensJohnson sydrome, nystagmus,
nausea, vomiting, drowsiness,
coarsening facial features, blood
dyscrasias

AMYOTROPHIC LATERAL SCLEROSIS. FEBRUARY 2009


1. This is part of Motor neuron disease
2. This is a progressive disorder of unknown cause, in which there is
degeneration of motor neurons in the spinal cord and cranial nerve nuclei,
and of pyramidal neurons in the motor cortex.
3. Between 5 and 10% of cases are familial, and in 20% of such families the
disease is caused by a mutation in the superoxide dismutase (SOD1) gene.
For the remaining 95%, possible causes include viral infection, trauma,
exposure to toxins and electric shock, but no sound evidence exists to
support any of these.
4. The prevalence of the disease is about 5/100 000. Patients present with a
combination of lower and upper motor neuron signs without sensory
involvement. The presence of brisk reflexes in wasted fasciculating limb
muscles is typical.
Clinical types of MND:
1. Progressive muscular atrophy
a. Predominantly spinal motor neurons affected
b. Weakness and wasting of distal limb muscles at first
c. Fasciculation in muscles
d. Tendon reflexes may be absent
2. Progressive bulbar palsy
a. Early involvement of tongue, palate and pharyngeal muscles
b. Dysarthria/dysphagia
c. Wasting and fasciculation of tongue
d. Pyramidal signs may also be present
3. Amyotrophic lateral sclerosis
a. Combination of distal and proximal muscle-wasting and weakness,
fasciculation
b. Spasticity, exaggerated reflexes, extensor plantars
c. Bulbar and pseudobulbar palsy follow eventually
d. Pyramidal tract features may predominate
Investigations:
1. Electromyography helps to confirm the presence of fasciculation and
denervation, and is particularly helpful when pyramidal features predominate.
2. Sensory nerve conduction and motor conduction studies are normal but there
may be some reduction in amplitude of action potentials due to loss of axons.
3. Spinal imaging and brain scanning may be necessary to exclude focal spinal
or cerebral disease.
4. CSF examination is usually normal, though a slight elevation in protein
concentration may be found.

61
Management:
1. The glutamate antagonist, riluzole, has been shown to have a small effect in
prolonging life expectancy by about 2 months.
2. Other agents such as nerve growth factor show promise.
3. Psychological and physical support, with help from occupational and speech
therapists and physiotherapists, is essential to maintain the patients quality
of life.
4. Mechanical aids such as splints, walking aids, wheelchairs and communication
devices all help to reduce handicap.

VERTIGO:
1. Definition:
Vertigo is defined as an abnormal perception of movement of the
environment and occurs because of an abnormality in sensory
information from the eyes, limb proprioception and the vestibular
system about a persons position in space.
2. There are four types of dizziness: vertigo, lightheadedness, presyncope,
and dysequilibrium. The most prevalent type is vertigo (i.e., false sense
of motion),
3. Vertigo caused by:
a. labyrinthine disorders
b. central (brain-stem) disorders
c. Vestibular disorders:
i. labyrinthitis,
ii. benign paroxysmal positional vertigo
iii. Mnires disease.
Differential Diagnosis:
1. Peripheral causes
a. Acute labyrinthitis: Inflammation of the labyrinthine organs caused by
viral or bacterial infection
b. Acute vestibular neuronitis (vestibular neuritis): Inflammation of the
vestibular nerve, usually caused by viral infection
c. Benign positional paroxysmal vertigo (benign positional vertigo):
Transient episodes of vertigo caused by stimulation of vestibular sense
organs by canalith; affects middle-age and older patients; affects twice
as many women as men
d. Cholesteatoma: Cyst-like lesion filled with keratin debris, most often
involving the middle ear and mastoid
e. Herpes zoster oticus (Ramsay Hunt syndrome): Vesicular eruption
affecting the ear; caused by reactivation of the varicellazoster virus
f. Mnires disease (Mnires syndrome, endolymphatic hydrops):
Recurrent episodes of vertigo, hearing loss, tinnitus, or aural fullness
caused by increased volume of endolymph in the semicircular canals
g. Otosclerosis: Abnormal growth of bone in the middle ear, leading to
immobilization of the bones of conduction and a conductive hearing
loss; this process also may affect the cochlea, leading to tinnitus,
vertigo, and sensorineural hearing loss
2. Central causes
h. Cerebellopontine angle tumor: Vestibular schwannoma (i.e., acoustic
neuroma) as well as infratentorial ependymoma, brainstem glioma,
medulloblastoma, or neurofibromatosis
1

62
i.

Cerebrovascular disease such as transient ischemic attack or stroke:


Arterial occlusion causing cerebral ischemia or infarction, especially if
affecting the vertebrobasilar system
j. Migraine: Episodic headaches, usually unilateral, with throbbing
accompanied by other symptoms such as nausea, vomiting,
photophobia, or phonophobia; may be preceded by aura
k. Multiple sclerosis: Demyelinization of white matter in the central
nervous system
3. Other causes
l. Cervical vertigo: Vertigo triggered by somatosensory input from head
and neck movements
m. Drug-induced vertigo: Adverse reaction to medications like alcohol,
aminoglycosides, anticonvulsants [phenytoin (Dilantin)], antidepressants, antihypertensives, barbiturates, cocaine, diuretics
[furosemide (Lasix)], nitroglycerin, quinine, salicylates,
sedatives/hypnotics)
n. Psychological: Mood, anxiety, somatization, personality, or alcohol
abuse disorders
4. Ninety-three percent of primary care patients with vertigo have benign
paroxysmal positional vertigo (BPPV), acute vestibular neuronitis, or Mnires
disease.
7

Distinguishing Characteristics of Peripheral vs. Central Causes of


Vertigo
FEATURE
PERIPHERAL VERTIGO CENTRAL VERTIGO

1.

2.

3.
4.

Nausea, vomiting

May be severe

Varies

Hearing loss,
tinnitus

Common

Rare

Latency following
provocative
diagnostic
maneuver

Longer (up to 20
seconds)

Shorter (up to 5 seconds)

VESTIBULAR NEURONITIS OR LABYRINTHITIS


This condition, also known as vestibular neuronitis, is the most common
cause of severe vertigo, but the cause is unknown. It usually presents in
the third or fourth decade as severe vertigo, with vomiting but no tinnitus
or deafness. It often starts on waking, and patients are usually bedbound
by the vertigo for the first few days.
The vertigo usually settles over 34 days, though head movement may
continue to provoke transient symptoms (positional vertigo) for some time
and it may take a month or so before the patient feels back to normal.
During the acute attack, nystagmus will be present but this typically
disappears after a few days.
Symptomatic relief can be achieved with antihistamines such as
cinnarizine, prochlorperazine, and betahistine. These agents should not be

63
used for long-term treatment of persistent vertigo as they are ineffective
and actually delay recovery in many patients.
5. Patients with intractable symptoms may need referral to an ENT specialist
for assessment, but input from an experienced vestibular physiotherapist
is invaluable.
Benign paroxysmal positional vertigo
1. This is due to the presence of otolithic debris from the saccule or utricle
affecting the free flow of endolymph in the semicircular canals
(cupulolithiasis).
2. It may follow head injury, but is more common in older patients
3. Each attack of vertigo lasts seconds but patients often become very
distressed and reluctant to move their head, and this can produce a
muscle tension-type headache.
4. Secondary hyperventilation attacks and associated depressive features are
also common.
5. The diagnosis can be confirmed by the Hallpike manoeuvre to
demonstrate positional nystagmus. Although this test is often difficult to
perform adequately as patients are frightened by the vertigo, it is useful
since it reassures the patient that the doctor knows what is going on.
6. Treatment comprises explanation, along with vestibular exercises designed
to send the otolithic debris back from semicircular canal to saccule or
utricle (such as the Epley manoeuvre) and/or to re-educate the brain to
cope with the inappropriate signals from the labyrinth such as Cawthorne
Cooksey exercises).
7. The Epley manoeuvre starts with the patient sitting upright on an
examination couch. They are asked to lie back with their head turned at a
45 angle to the affected side, and to hold this position for 3060 seconds.
They should then be asked to turn their head 90 to the opposite side and
remain in that position for 3060 seconds. Next, they should be asked to
roll on to their side in the direction they are facing, so that their face is
pointing towards the floor, and hold that position for 3060 seconds before
sitting up once again.
8. Cawthorne-Cooksey exercises involve asking the patient to perform a
series of eye, head and whole-body movements whilst lying down, sitting
and standing. Antihistamines can be tried if the condition fails to respond
to the above measures.
Mnires disease
1. This is due to an abnormality of the endolymph which gives rise to
paroxysms comprising a sensation of fullness in the ear followed by
headache, tinnitus and profound vertigo, each attack typically lasting
about 4 hours. In between attacks, patients hearing may be distorted
(typically low-tone deafness).
2. While sensorineural hearing loss may be demonstrable, bedside
examination is typically otherwise normal in between attacks, but
audiometry is usually abnormal.
3. Management is as described for labyrinthitis.

GENERAL PARALYSIS OF INSANE:

64
1. Neurosyphilis may present as an acute or chronic process and may involve
the meninges, blood vessels and/or parenchyma of the brain and spinal
cord.
2. syphilis is now most commonly seen in patients with AIDS.
3. Pathology:
a. General paresis is a chronic inflammation of the brain and its lining
(meningoencephalitis) that occurs in the late stages of syphilis.
b. Degeneration in cerebral cortex/cerebral atrophy
c. Thickened meninges
4. Clinical features
a. The individual or his or her family may report irritability, memory
loss, personality changes, impaired concentration, carelessness,
headache, and insomnia.
b. In the later stages muscle weakness, defective judgment,
depression, psychosis, dementia, confusion, disorientation,
paranoia, and seizures may occur.
c. The final stage of the disease may include frequent seizures,
incontinence, and recurrent strokes.
5. Physical exam:
a. Individuals may present with tremors of the lips, tongue, and
fingers; slurred speech; difficulty writing; unsteadiness; decreased
muscle tone; muscle weakness; lack of facial expressions; and small,
nonreactive pupils.
b. Other conditions associated with late-stage syphilis infection may
also be present, such as tabes dorsalis, another disease that affects
the nervous system.
c. Individuals may also exhibit psychological instability that mimics
schizophrenia, mania, paranoia, or depression.
6. Tests:
a. Non-treponemal (non-specific) tests: Venereal Diseases
Research Laboratory (VDRL) test; Rapid plasma reagin (RPR) test
b. Treponemal (specific) antibody tests:
i. Treponemal antigen-based enzyme immunoassay (EIA) for IgG
and IgM
ii. T. pallidum haemagglutination assay (TPHA)
iii. T. pallidum particle agglutination assay (TPPA)
iv. Fluorescent treponemal antibody-absorbed (FTA-ABS) test

v.

CSF examination is essential if neurological involvement is


suspected. Active disease is suggested by an elevated cell

65
count, usually lymphocytic, and the protein content may be
elevated to 0.51.0 g/L with an increased gamma globulin
fraction. Serological tests in the CSF are usually positive, but
progressive disease can occur with negative CSF serology.
7. Treatment:
a. The injection of procaine benzylpenicillin (procaine penicillin) and
probenecid for 17 days is essential in the treatment of neurosyphilis
of all types
b. Emergency treatment of seizures may be required. Antiseizure
medicine may be prescribed for controlling seizures.
c. Assistance or supervision with activities such as eating and dressing
may be required if individuals can no longer care for themselves.
d. Physical or occupational therapy may be required for those with
muscle weakness.
e. General paresis can take away individuals' ability to care for
themselves and communicate or interact with others. Progressive
disability is likely, although treatment can reduce (but not
necessarily stop) the progression of nerve damage. Damage already
done to organs cannot be reversed.
8. Prognosis: If untreated, this condition is fatal, with death typically occurring
within 5 to 6 years following the development of symptoms.

NEURALGIC AMYOTROPHY - APR 97


i.

ii.
iii.
iv.
v.

vi.
vii.

Neuralgic amyotrophy (NA) is an uncommon disorder of the peripheral


nervous system characterized by the sudden onset of extreme pain in the
upper extremity followed by motor weakness and atrophy and a slow
recovery in months to years.
It includes both an idiopathic (INA, also known as Parsonage-Turner
syndrome) and hereditary (HNA) form.
The minimum incidence of NA is estimated at 1/50,000
NA can occur at any age but is most frequently seen in those between the
3rd-7th decades of life and is more frequent in men.
The classic presentation manifests with sudden onset of aching, burning,
or stabbing pains, most often in the shoulders, neck, and/or arm region,
showing an upper brachial plexus distribution. Weakness in the
periscapular and periglenohumeral muscles follows hours to weeks after
the initial onset of pain.
These pains are usually relentless, worse at night and last for about 3
weeks.
Other manifestations may occur due to the involvement of nerves outside
the brachial plexus such as the lumbosacral plexus or phrenic nerves.

66
viii.
ix.

x.
xi.

xii.
xiii.

xiv.

xv.

xvi.

Certain patients with HNA display characteristic physical features (ex.


hypotelorism, slanted eyes, epicanthal folds, oval face, cleft palate).
Some patients experience a relapsing/remitting course with symptom-free
intervals while others have an incomplete recovery with persisting
neurologic deficit.
Recurrences can occur and persistent musculoskeletal pain develops in
2/3 of patients.
The exact etiology is unknown but genetic, autoimmune and external
factors are thought to play a role. The brachial plexus has an underlying
predisposition to mechanical injury and attacks caused by an immunemediated response in this nerve. A viral infection or immunization can
precede NA. Bacterial and parasitic infections, surgery, anesthesia,
rheumatic disease, trauma, pregnancy and childbirth have all been
implicated as possible contributing factors of NA attacks.
HNA is associated with a point mutation or duplication of the susceptibility
gene on chromosome 17
Diagnosis is based on the typical clinical features and the exclusion of
other disorders (ex. neuroborreliosis, cervical radiculopathy or Pancoast
syndrome) using laboratory tests, electromyography and imaging of the
cervical spine and brachial plexus.
In those where HNA is suspected, a molecular genetic test can be used to
identify a SEPT9 mutation, but since HNA is genetically heterogeneous, a
negative test does not exclude the diagnosis.
Treatment:
a. In the acute stage involves pain management, relying mostly on a
combination of long-acting opioids and nonsteroidal antiinflammatory drugs.
b. Oral prednisone decreases the duration of pain and accelerates
recovery in some patients if given in the first weeks of an attack. A
daily dose of 1mg/kg for 1 week which is tapered during the 2nd
week is recommended. Follow-up is recommended every 6 months.
c. Rehabilitation therapy is now considered important and excessive
strain to the affected area should be avoided.
The prognosis:
a. is variable but thought to be good with patients recovering 70-90%
of their previous health after 1-2 years. However, many are left with
exercise intolerance and poor muscular coordination in the affected
and compensating muscles.
b. Quality of life can be affected in those with an incomplete recovery
but early diagnosis and proper treatment increase the chances of a
full functional recovery.

LOSS OF MEMORY:
1. Loss of memory for a period of time may be due to:
a. a transient toxic confusional state,
b. a psychological fugue state,
c. the post-ictal period after seizure or
d. the syndrome known as transient global amnesia. These are usually
distinguished on the basis of the history.

67
2. A period of amnesia often follows either a complex partial or a generalised
seizure, and this may cause diagnostic confusion if the seizure was not
witnessedfor example, if it occurred in sleep.
3. Symptoms of amnesia:
a. Asking the same questions repeatedly
b. Forgetting common words when speaking
c. Mixing words up saying "bed" instead of "table," for example
d. Taking longer to complete familiar tasks, such as following a recipe
e. Misplacing items in inappropriate places, such as putting a wallet in a
kitchen drawer
f. Getting lost while walking or driving around a familiar neighborhood
g. Undergoing sudden changes in mood or behavior for no apparent
reason
h. Becoming less able to follow directions
4. Causes of memory loss:
a. Wernicke-Korsakoff syndrome,
b. carbon monoxide poisoning,
c. herpes encephalitis and other central nervous system infections,
hypoxic and other acquired brain injuries,
d. stroke,
e. brain tumors, and neurosurgical resections
f. conversion disorder,
g. posttraumatic stress disorder (PTSD),
h. dissociative disorders,
i. malingering.
j. electroconvulsive therapy
Transient global amnesia
1. This is a syndrome affecting predominantly middle aged patients
2. There is an abrupt loss of memory function which lasts for a period of a
few hours
3. During the attack there is retrograde amnesia for the events of the past
few weeks.
4. After 46 hours, memory function and behaviour return to normal but the
patient has persitent, complete amnesia for the duration of the attack
itself.
5. There is no associated with seizures and, unlike epileptic amnesia,
transient global amnesia tends not to recur.
6. There is no vascular aetiology. Transient global amnesia is thought to be
due to a benign process similar to that causing a migrainous aura,
occurring in the hippocampus.
7. The patient has no physical signs and further investigation may not be
needed if epilepsy can be excluded.
Persistent amnesia
1. Patients with persistent memory disturbance should be assessed to
exclude serious neurological disease.
2. memory problems might have been observed by relatives or colleagues
3. Problems of concentration must be distinguished from loss memory, as
concentration difficulties are due to underlying depressive or anxiety
disorders.

68
4. Complaints of getting lost or of forgetting to switch off burners on the
kitchen stove are more likely to be significant than simply forgetting
names.
5. Causes:
a. Korsakoffs syndrome
b. Bilateral temporal lobe damage
c. Different types of dementia.
i. Progressive deterioration over many months suggests the
possibility of an underlying dementia
ii. A family history of a memory disorder such as dementia .
d. Depression:
i. It is important to look for features of depression in patients
with memory loss for two reasons.
ii. Depression can present as a pseudo-dementia with
concentration and memory impairment as a dominant
feature, and this is often reversible with antidepressant
medication.
iii. Second, many patients with dementia may develop
depression in the early stages of their illness, and this is also
potentially treatable.
6. Treatment is for the primary cause such as dementia , depression of
organic pathology

PONTINE HAEMORRHAGE
A pontine haemorrhage, a form of intracranial haemorrhage, is most
commonly due to long standing poorly controlled chronic hypertension. It carries
a very poor prognosis.
Epidemiology
Primary pontine haemorrhages account for approximately 10% of all
intracranial haemorrhages. Typically patients are elderly with a long history of
poorly controlled hypertension. Haemorrhage into the pons can of course also
be secondary to underlying lesions including:
1. Vascular malformations
a. cavernous malformations
b. arteriovenous malformations
2. Tumours
a. Neuroepithelial (primary) brain tumours
b. metastases
3. Downward herniation (duret haemorrhages)
4. Supratentorial surgery (remote haemorrhage)

Clinical presentation
Patients present with sudden and precipitous neurological deficit. Depending
on the speed at which the haematoma enlarges and the exact location,
presentation may include 1-2:
1. decreased level of consciousness : most common

69
long tract signs including tetraparesis
cranial nerve palsies
seizures
Cheyne-Stokes respiration
Pathology
2.
3.
4.
5.

1. As is the case with penetrating arteries into the basal ganglia, the
penetrating arteries from the basilar artery extending into the pons are
subject to lipohyalinosis as a result of poorly controlled hypertension.
2. This renders the vessel wall prone to rupture. The larger paramedian
perforators are more commonly the culprit vessels.
Radiographic features
CT
1. CT of the brain is usually the first, and often the only, investigation
obtained upon presentation. Features typical on an acute intraparenchymal
haemorrhage are noted, usually located centrally within the pons
2. In patients who have small volume bleeds and who are thought to possibly
have an underlying lesion, MRI may be use (e.g. identification of a vascular
malformation).
Treatment and prognosis
1. Pontine haemorrhages have a poor prognosis, with large bleeds being
almost universally fatal. Open surgical evacuation of the clot is usually not
performed, although stereotactic clot aspiration has been advocated by
some 5.
2. In smaller haemorrhages, medical management and treatment of
hydrocephalus with extraventricular drains may be life saving, often
however with significant residual handicap.
3. Overall mortality is around 40-50%, with the overall volume of the bleed
being related to outcome 2.
Differential diagnosis
1. The main differential is between a primary pontine haemorrhage and
haemorrhages resulting from underlying lesions (see above). Usually
patients present suddenly with severely impairment and the diagnosis is
not difficult to make.
2. In patients where the presentation is not known (e.g. in the exam setting)
it is worth considering:
1. Un-ruptured / asymptomatic vascular malformations

70
a. Asymptomatic cavernous malformations (these can

periodically have small bleeds resulting in repeated


symptoms)
b. Arteriovenous malformation
Usually little if any mass effect
Serpentine irregular density isodense to
intravascular blood elsewhere
c. Developmental venous anomaly
Linear, no mass effect
Density isodense to intravascular blood elsewhere
2. Haemorrhagic metastases
a. Melanoma, renal cell carcinoma, thyroid carcinoma etc...

Anda mungkin juga menyukai